Наука

Ответить в тред Ответить в тред
Check this out!
Тред тупых вопросов. Прошлый хер знает где Тред тупых вопросов 14/01/21 Чтв 02:25:25 5264191
16082642596450.png 409Кб, 680x591
680x591
Тред тупых вопросов. Прошлый хер знает где
Аноним 14/01/21 Чтв 09:11:56 5264372
Что такое электромагнитный потенциал, если сила тока - количество заряда на время?
Аноним 14/01/21 Чтв 11:42:14 5264463
>>526437
Изначально это потенциальная энергия взаимодействующих зарядов. Если обобщить, то это как вообще электромагнитное поле физически реализуется.
Ток, вообще плотность тока - векторная величина, это как электромагнитное поле видит другие сущности.
Аноним 14/01/21 Чтв 16:54:57 5264934
>>526437
ЭМ Потенциал точки - это количество потенциальной энергии единичного заряда, помещенного в эту точку. Абсолютная аналогия с механической потенциальной энергией. Ксли ты разместишь кирпич в метре от земли, то у него будет дохуя потенциальной энергии, засчет которой он и полетит вниз. Если ты поместишь электрон на каком-то расстоянии от протона, то у электрона будет дохуя потенциальной энергии, засчет которой он полетит к нему. Энергия, которой он будет обладать, будучи изначально неподвижным, в этой точке, и есть электрический потенциал. Магнитный потенциал - то же самое, но для движущихся зарядов. То есть если заряд движется и проходит через какую-то точку в магнитном поле, на него начнет в этой точке действовать сила Лоренца, увлекая в каком-то направлении. Эта сила на его суммарное перемещение в поле после этого момента и будет потенциальной энергией минус дельта кинетической энергии в начале и в конце. Все вместе - ЭМ потенциал в данной точке.
Аноним 14/01/21 Чтв 17:22:01 5264965
>>526493
А тепеть объясни причем электромагнитный потенциал дифференциальные формы.
Аноним 15/01/21 Птн 13:24:35 5265816
>>526496
Не совсем понимаю, о чем ты. Формулу покажи, к которой ты задаешь вопрос.
Аноним 15/01/21 Птн 23:26:21 5266067
Почему бабы, когда воняют, то воняют не как мужик — пОтом и нечищеными зубами, а какой-то вообще уникальной неидентифицируемой вонью, которая вселяет инфернальное недоумение и страх? Это из-за гормонов пот так по-другому душнит, или эт запах пизды?
Тред тупых вопросов 16/01/21 Суб 01:20:36 5266088
>>526606
менструальная кровь
Аноним 17/01/21 Вск 16:32:29 5267149
yadernaya-reakt[...].jpg 27Кб, 400x312
400x312
Посоны, реально ли снизить критическую массу деления нестабильного изотопа путём увеличения температуры и плотности этого изотопа? Что если например нестабильные изотопы радона залить в токамак, ну и ясен пень ионизировать, сжать и нагреть? Начнётся ли ядерное деление при меньшей массе? Ведь тогда можно сделать домашний ядерный реактор размером с электроплиту. Ну или ядерный ракетный двигатель на жидких или газообразных нестабильных изотопах.
Аноним 17/01/21 Вск 16:39:53 52671610
Просто вот поглядел видосик и подумал, а зачем делать синтез когда с распадом всё куда проще, величины температуры и плотности меньше, а выхлоп есть и ещё какой. https://www.youtube.com/watch?v=ZpdeJGMuNnc
>>526714
Аноним 17/01/21 Вск 16:40:47 52671811
>>526714
>и плотности этого изотопа
У тебя шиза пздц, ты то про увеличение плотности, то про превращение изотопа в газ
Как ты собрался сжимать твёрдые ядерные материалы? Имплозией? в домашних условиях?
С газом радиоактивным работать вообще охуеть какая идея.
Аноним 17/01/21 Вск 16:42:13 52672112
>>526718
>Как ты собрался сжимать твёрдые ядерные материалы? Имплозией? в домашних условиях?
ну и ясен пень ионизировать, сжать и нагреть
>С газом радиоактивным работать вообще охуеть какая идея.
Да охуенная идея. В совке вон радоновые ванны практиковали для больных.
Аноним 17/01/21 Вск 16:44:22 52672313
>>526718
Ты не понел вообще. Я сказал - взять нестабильный изотоп в жидком или газообразном состоянии (например изотоп радона), залить его в токамак и сделать с ним тоже самое что делают с дейтерием и тритием в токамаке ёпт. Токамак ионизирует, сжимает и нагревает эту поебень.
Аноним 17/01/21 Вск 16:47:20 52672414
радоновая ваннай.png 332Кб, 1896x957
1896x957
>>526721
>В совке вон радоновые ванны практиковали для больных.
В Питере до сих пор этим занимаются
Аноним 17/01/21 Вск 16:52:22 52672615
>>526718
Иными словами сделать распад в установке, которая предназначена для синтеза.
Аноним 17/01/21 Вск 16:52:48 52672716
Что такое поле?
Аноним 17/01/21 Вск 16:55:52 52672817
>>526727
И что такое квант поля?
Аноним 17/01/21 Вск 16:57:13 52672918
>>526728
>квант
Минимально возможная величина которую можно посчитать
Аноним 17/01/21 Вск 16:58:20 52673019
>>526419 (OP)
Сможет ли жить человек во вселенной, где скорость света в вакууме не такая, как в нашей вселенной? Вопрос по "черным доменам" Лю Цысиня
Аноним 17/01/21 Вск 17:10:13 52673220
>>526729
Посчитать? А как?
Аноним 17/01/21 Вск 17:13:33 52673321
Почему скорость света в вакууме 299 792 458 метров в секунду,
а скорость света в конденсате Бозе-Эйнштейна 0,2 мм в секунду?
Тред тупых вопросов 17/01/21 Вск 18:32:48 52673622
16105741856480.png 17Кб, 800x600
800x600
Если взять группу шимпанзе в естественной среде и раздать им копья мачете, как будет развиваться картина?
Аноним 17/01/21 Вск 18:37:14 52673723
>>526736
Интересный эксперимент!
Аноним 17/01/21 Вск 18:40:22 52673824
>>526733
Сокрость света не 299 792 458 метров в секунду, это 1 секунда равна тому за сколько свет проходит это метр равен тому сколько свет проходит за 37_849_9_/_)$)3)$)$) колебаний атома цезия или какой-то другой хуйни.
Аноним 17/01/21 Вск 18:43:37 52674025
>>526732
Там с проваливанием в чёрную дыру чё-то.
Типа такое маленькое, что если бы было ещё меньше было бы чёрной дырой.
Аноним 17/01/21 Вск 18:47:39 52674126
>>526723
>В настоящее время известно 39 изотопов радона с массовыми числами от 193 до 231 и периодами полураспада от 0,3 нс до 3,8 сут.
Только в попу себе можешь закачать такое "топливо".
Тред тупых вопросов 17/01/21 Вск 18:55:13 52674227
>>526737
Шимпанзе вечно ведут войны между племенами, мне интересно, вооруженная группа будет доминировать и расти, или все переубивают друг друга за неделю
Аноним 17/01/21 Вск 20:41:28 52675228
>>526742
О, я кстати вспомнил что видел какой-то видос давно, типа натуралистическая программа, там рассказывалось что-то типа "напали на соседнее племя, похитили убили и съели несколько самцов, самок забрали в своё племя"
збс, всё как у людей
Аноним 17/01/21 Вск 21:56:23 52675329
>>526727
Гладкая функция от координат.
Аноним 17/01/21 Вск 22:27:02 52675430
Эксперты в фотонах ещё тут?


*монохроматические лазеры" или светодиоды они реально монохроматические, или там просто относительно узкий участок спектра они испускают?
Аноним 17/01/21 Вск 22:33:17 52675531
>>526754
*и вообще есть ли какие-то способы излучать реально чисто 1 фотон
Аноним 17/01/21 Вск 22:40:13 52675632
>>526754
Лазер дает очень много фотонов с одной длиной волны и фазой. Прям совсем точно.
Со светодиодами свои особенности, но полупроводниковые лазеры таки лазеры.
Аноним 17/01/21 Вск 22:45:38 52675733
>>526756
Вот прямо чисто ровно одна длина волны?
Я думал у них принцип работы как у светодиодов, а насчёт "спектра монохроматического светодиода" гугл какую-то хуйню выдаёт, походу нету в них никакой монохроматичности, просто какое-то нормальное распределение на весь их цвет свечения.
Аноним 17/01/21 Вск 22:49:08 52675834
>>526755
Ну способов много, только нет толковых, чтоб там всякие квантовые компьютеры делать.
Аноним 17/01/21 Вск 22:50:32 52675935
>>526758
Не, мне не для этого, а для каантовых компьютеров это разве надо?
Аноним 17/01/21 Вск 22:51:17 52676036
>>526758
>Ну способов много
Какие есть?
Аноним 17/01/21 Вск 23:10:55 52676137
>>526757
Ну лазер должен выдать строго одну линию. Причем по расползанию спектра можно померить несовершенство детектора.
Есть просто светдиоды, хоть они могут называться монохроматическими, но они не лазеры.
Лазерные диоды обычно называют просто полупроводниковыми лазерами. Строго говоря это даже не совсем светодиоды. Эмиссия происходит по разным процессам.

В быту самый простой способ проверить лазер или нет - посмотреть на источник через компакт диск. Если на нем интерференция размазывается в радугу, то это не лазер.

>>526759
Для квантовых компьютеров нужен источник перепутанных фотонов. Короче нужно излучать строго 2(-3-до бесконечности) запутанных фотона. С одиночным фотоном только один сраный кубит выходить и все вычисление сводятся к играм поляризаторами на коленке.
>>526760
Ловишь возбужденный атом в ловушку и пинаешь его(выпускаешь из ловушки или сторонней частицей). Можно такое делать на дефектах в кристаллах(например на центрах окраски) или всякие мета-стабильные состояния дырок и электронов в твердом теле.
Аноним 17/01/21 Вск 23:22:30 52676338
>>526761
>Ловишь возбужденный атом в ловушку и пинаешь его
Мне тут раньше говорили что так испускается пакет фотонов разных длинн волн, который как бы все считают "одним" фотоном
Хотя на самом деле это как понял куча разных фотонов разных длинн волн.
Аноним 17/01/21 Вск 23:27:39 52676439
Но вообще хз как можно выдать строго одну длину волны
это значит что она выдаётся с бесконечной точностью, а это анриал

отчего она там зависит? от энергии перехода? энергия точно не может быть до бесконечности одинаковой каждый раз
Аноним 18/01/21 Пнд 00:06:47 52676640
>>526763
Че то ты не правильно понял.
Одиночный фотон это пакет или совокупность определенных классических электромагнитных волн, каждой которой приписали свой "вес". Совокупность этих волн образует линейное пространство - одну волн можно рассмотреть как совокупность двух других. Когда проходит измерение детектором для нашей реальности становиться актуальна только одна волна из всего пакета волн. Важный момент, для того чтоб измерить фотон, нужно с ним что-то сделать. Возмущение фотона ведет в тому, что у нас может сильно поменяться какие волны будут выпадать. Под возмущением подразумевается, как он создается, что с ним может произойти по пути от источника до детектора + сам детектор. Существует способ формализовать возмущение и найти его влияние на конечный наблюдаемые волны.
Лазер же источник одинаковых фотонов, или куча всяких фотонов испытывает одно и тоже возмущение.
Аноним 18/01/21 Пнд 03:11:35 52677441
>>526738
>это метр равен тому сколько свет проходит за
в какой среде?
Аноним 18/01/21 Пнд 03:12:13 52677542
>>526740
>Типа такое маленькое, что если бы было ещё меньше было бы чёрной дырой.
А при чем тут черные дыры вообще?
Аноним 18/01/21 Пнд 03:13:38 52677643
>>526754
> они реально монохроматические
Да. Монохроматические, когерентные, поляризованные и узконаправленные.
Аноним 18/01/21 Пнд 03:14:56 52677744
>>526764
>а это анриал
Чо так?
Аноним 18/01/21 Пнд 03:15:34 52677845
>>526766
>Че то ты не правильно понял.
>Одиночный фотон это пакет или совокупность определенных классических электромагнитных волн
Чел просто думает, что фотон это такая частица, ну типа маленький шарик.
Аноним 18/01/21 Пнд 07:40:46 52679046
>>526778
А ты явно думаешь много маняфантазий на счёт него.
Тред тупых вопросов 18/01/21 Пнд 11:30:26 52679847
>>526752
А еще некоторые ебут слабых самцов в жопы, но это не шимпанзе
Аноним 18/01/21 Пнд 11:32:20 52679948
Тред тупых вопросов 18/01/21 Пнд 11:36:23 52680149
m85AnEzz.png 384Кб, 455x397
455x397
>>526799
>репорт щитпостинга
Аноним 18/01/21 Пнд 16:07:50 52681050
>>526714
Во-первых радон не делится. Для деления подходят только U-233, U-235, Pu-239. Все остальные изотопы и элементы либо обладают маленькими сечениями деления, либо хуй их изолируешь от других изотопов (Pu-241, например)

Да, температура влияет на реактивность реактора, но в другую сторону. При увеличении температуры топлива резонансы поглощения нейтронов на уране-38 уширяются из-за Доплер-эффекта, он начинает жрать больше нейтронов -> меньше нейтронов остаётся делящимся нуклидам и реактивность падает.

Плотность также влияет на критичность. Больше плотность -> больше реактивность, так устроена атомная бомба.

Домашние реакторы размером с электроплиту уже есть (Ромашка, корабельные реакторы), однако они работают на промежуточных/быстрых нейтронах, поэтому нуждаются в гораздо более высоком обогащении.

Проекты ядерных двигателей, где деление происходит прямо в сопле (Плутоний с водяным паром, который выступает в роли замедлителя, прокачиваются через сопло), мусолились в ХХ веке, однако любая ядерная делящаяся хуйня в космосе очень опасна.

>>526726
Бред.
Аноним 18/01/21 Пнд 16:47:45 52681251
010146008203169[...].png 81Кб, 2194x2862
2194x2862
>>526810
>Для деления подходят только U-233, U-235, Pu-239. Все остальные изотопы и элементы либо обладают маленькими сечениями деления
Аноним 18/01/21 Пнд 16:54:21 52681352
>>526812
И? Мб всю мою цитату скопируешь?

>Все остальные изотопы и элементы либо обладают маленькими сечениями деления, либо хуй их изолируешь от других изотопов

Большая часть нуклидов, которая есть на твоей табличке либо пиздец нестабильная (Фермий), либо пиздец фонящая(Уран-232), либо невозможно отделить от других изотопов (Все Pu после Pu-39).

Поэтому за пределами лабораторий ничего кроме урана, плутония и, прости господи, ТОРИЯ нахуй не нужно.
Аноним 20/01/21 Срд 16:42:21 52698253
Пацыки, такой вопрос, в КМ целая пачка уравнений, Шрёдингера, Дирака, Паули, КГФ и ещё черте что. Дирак - это ур-е на электрон или любой фермион, релятивистское, к тому же, Паули вроде то же, но нерелятивистское, КГФ - это вообще ур-е на свободное поле, а что же тогда Шрёдингер? Он не работает для электронов разве? Но ведь Эрвин проверял его на спектрах водорода, то есть на тех же электронах. Короче, вопрос такой: что не так со Шрёдингером, что он описывает и чего не описывает?
Аноним 20/01/21 Срд 17:24:03 52698554
>>526982
Есть 3 уравнения, которые называют уравнение шредингера:
1) Н|ψ> = E|ψ>
2) H|ψ> = i hbar d/dt|ψ>
3) H =P^2/2m + U и пункт 1
Первые 2 работают всегда. 3я только для бесспиновых нерелятивистских частица.
Аноним 20/01/21 Срд 22:10:45 52701655
Что сделают раньше: долетят до соседней звезды или добурят до ядра Земли?
Аноним 20/01/21 Срд 23:54:38 52702456
>>527016
Скорее первое. Все что ниже коры "жидкое" вроде как. Как ты жидкость бурить собрался
Аноним 21/01/21 Чтв 07:47:18 52703857
>>526419 (OP)
Возможно ли,
в детерминистичном и фаталистичном ебне,
запрограммировать,
возможность исключить,
вероятность не успеть среагировать на инфу актуальную,
получаемую из модели высокоточной,
которая может, блядь,
потерять актуальность - в процессе её извлечения?
Аноним 21/01/21 Чтв 11:17:54 52704658
>>526419 (OP)
Вы сами написали, что тред тупых вопросов, поэтому пинайте на себя


Почему никто не рассматривает глионы на роль гравитонов? Безмассовые частицы, как и гипотетический гравитон. Сильное взаимодействе между кварками резко ослабляеться на больших расстояниях, тогда как гравитация является слабой силой. И в сверхмассивных объектах, гипотетически, имеется большое колличество кварков и глионов, как раз для взаимного притяжения на больших расстояниях. То есть, гипотетически, на малых расстояниях глион как причина сильного ядерного взаемодействия, а на больших - гравитационного


Почему все так зациклились на абстракции "кривизны просторанства-времени", если гравитационное взаимодействие/поле отличается от электромагнитного только силой и участвующими частицами? да и другие взаимодействия имеют свои частицы-переносчики
Аноним 21/01/21 Чтв 11:20:23 52704759
21/01/21 Чтв 22:35:44 52707960
>>526736
>копья
Они вроде и сами палки собирают и пиздиют ими друг друга
Аноним 21/01/21 Чтв 23:41:56 52709261
>>527046
Никто не понимает, как квантовать гравитацию и получать эти пресловутые "гравитоны", потому что гравитация - это искривление пространства-времени массой содержащихся в нем объектов. В реальности не существует никакого "гравитационного поля" и "гравитационного взаимодействия" (Стандартная модель элементарных частиц недаром выделяет только три взаимодействия - сильное, слабое, электромагнитное. Гравитация Стандартной моделью вообще никак не затрагивается), ОТО в принципе не оперирует этими понятиями. Гравитация - это, как уже говорилось, искривление пространства-времени и ничего более.
Аноним 22/01/21 Птн 00:14:07 52709462
>>527079
Ну так смысл же в том, чтобы было превосходство в оружии, а палки любые макаки поднять могут.
Аноним 22/01/21 Птн 00:29:44 52709663
>>527046
>Почему все так зациклились на абстракции "кривизны просторанства-времени", если гравитационное взаимодействие/поле отличается от электромагнитного только силой и участвующими частицами? да и другие взаимодействия имеют свои частицы-переносчики
Скорее "переносчики взаимодействия частицы" это абстракция, причём самая кастыльная.
Вот в тупую "была частица, у неё изменились параметры, значит это летела частица, несла посылочку, в которой было всё что изменилось ровно на разность того что было и того как стало".

Это может быть норм для расчётов, ну просто условность, но явно не так как оно реально устроено.
Аноним 22/01/21 Птн 01:00:50 52709764
>>527092
>искревление пространства-времени и ничего более
Ну тогда получаеться, что и магнит тоже искревляет пространство-время, но не для всех частиц, кек

>частица
>условность

>кривизна самого бытия
>не условность

Вообще не могу понять эту логику математиков абстрактоцентристов
Аноним 22/01/21 Птн 01:01:57 52709865
Аноним 22/01/21 Птн 02:05:51 52710366
>>527097
>Вообще не могу понять эту логику математиков абстрактоцентристов
Ну про бытие это ты сам-себе нафантазировал, а в кривизне пространства что не так?
Время не нужно сюда приплетать
Аноним 22/01/21 Птн 02:15:50 52710567
>>527097
При чем тут частицы? И при чем тут магнит? Любая масса искривляет пространство-время - это и есть гравитация. Ты какую-то хуйню несешь.
Аноним 22/01/21 Птн 11:59:51 52712468
Теоретически, кто может создать установку для исследования влияния частоты внешнего поля на вещество?
Как пример эксперимент, который всем известен.
Берем воду, изучаем её поглощение в широком диапазоне частоты.
На выходе как минимум получаем СВЧ бытовую печь для потребителя.
Аноним 22/01/21 Птн 18:54:38 52715569
>>527105
>При чем тут частицы?
Вообще не причем. Забей, их же не существует, абстакция

>И при чем тут магнит?
У него есть притяжение? Есть. Значит и кривизна есть. У него еще и отталкивание есть, значит кривизна положительной может быть получается?

>Любая масса искривляет пространство-время - это и есть гравицая
А электро-магнитизм - это искривление пространства-времени на пол шишички получается?

>Ты какую-то хуйню несешь.
Глазки и ушки закрыл и стал бубнить "квантовик, ухади; квантовик, ухади"
Аноним 22/01/21 Птн 20:32:35 52715970
Вот есть курица, яйцо и петух. Неоплодотворенные яйца идут в пищу, оплодотворенные яйца идут в инкубатор. Чтобы одно отличить от другого есть овоскоп (хотя он вроде и чет другое ещё показывает). Вместо него можно использовать фонарик на телефоне и будет видно пустое ли яйцо, чтобы есть.

А без даже фонарика как раньше определяли? И что будет если есть оплодотворенное? Гроб кладбище пидор?

На дваче может за такой вопрос не засмеют, вот здесь и спрашиваю.
Аноним 22/01/21 Птн 20:34:02 52716071
>>527159
И ещё, наверное, тогда вопрос про петуха. Как так получается, что хотя он курицу и приходует постоянно, но при этом она несет неоплодотворенные? Не хватает что ли?
Аноним 22/01/21 Птн 21:33:26 52716472
Бухгалтеры есть тут? Можете помочь, пожалуйста?
Аноним 22/01/21 Птн 22:11:27 52716573
>>527124
Этим спектроскопия занимается, для разных частот разные установки
Аноним 23/01/21 Суб 00:20:45 52716874
>>527155
Блять, ты в бреду, иди проспись. При чем тут вообще электромагнетизм? Какая нахуй кривизна магнита? Что ты несешь-то, поехавший блять? Меня твои истории просто доебали, я уже не могу их слушать. Быстро иди учи ОТО. Только протрезвей для начала.
Аноним 23/01/21 Суб 01:33:17 52717075
Я начал готовиться к ЕГЭ с нуля, хочу пойти на "лечебное дело", возраст: 25 лет, какие у меня шансы делать науку, если я буду прилежно усваивать учебный материал? Планирую только качаться и впитывать знания.
Аноним 23/01/21 Суб 02:36:29 52717276
Аноним 23/01/21 Суб 02:40:03 52717377
>>527159
Ты вкурсе что есть мало того что такой деликатес-почти созревший эмбрион в яйце, его едят
Но и такое-почти созревший эмбрион, затем убитый и немного подтухший?
Разбивают, и выпивают эту смесь из костей, кожи, крови, кишок эмбриона только сформировавшихся, небольших только начавших появляться перьев, и остатков белка и желтка
И это всё немного подгнившее и подбродившее
Аноним 23/01/21 Суб 08:21:46 52717978
>>527168
>Блять, ты в бреду

>частицы абстракция
>в бреду я...

>При чем тут вообще электромагнетизм?
Поле есть? Есть. Притяжение есть? Есть. Вопросы?

>Какая нахуй кривизна магнита?
Не, ну на пол шишечки. Притяжение же есть

>Меня твои истории просто доебали
А меня твои

>Быстро иди учи ОТО
Я +- знаю ОТО, и не отрицаю, что по нему можно и удобно делать вычисления в астрофизике. Меня вымораживают не расчеты, а абстрактофилия
Я бы послал тебя учить квантмех, но мне одинаково будешь ты его знать или нет
Аноним 23/01/21 Суб 08:56:08 52718279
>>526754
Очень очень узкий. Идеально не выйдет получить, ну там дохуя проблем это вызывают.
Аноним 23/01/21 Суб 15:52:56 52719180
Почему вращательное движение абсолютно? А поступательное движение, которое можно рассматривать как частный случай вращательного (с бесконечно удаленной от тела осью вращения) - относительно.
Аноним 23/01/21 Суб 16:02:55 52719281
>>527179
>Поле есть? Есть
Есть модель для расчётов, шизик.
Аноним 23/01/21 Суб 16:07:14 52719482
>>527191
С этой относительностью вообще какая-то мутная хуйня
Для фотона вроде бы как времени не существует, в том смысле что он для себя самого существует 0 времени
Но вот для стороннего наблюдателя он существует, имеет частоту и прочие другие свойства
Такая же хуйня с движущимися на встречу фотонами
Сложение скоростей, вся хуйня, будет скорость их относительно друг-друга равна скорости света
Но для стороннего наблюдателя, наблюдающего за их движениям, они сблизятся на расстояние со скоростью 2с за единицу времени за которую свет в одном направлении проходит 1с.
Аноним 23/01/21 Суб 16:50:13 52719583
>>527194
Это конечно все хорошо, но хотелось бы и с точки зрения классической механики как-то это осмыслить. Почему вращающаяся система отсчета является неинерциальной? Какая связь между равномерным вращением и поступательным ускорением?
Аноним 23/01/21 Суб 17:02:48 52719684
>>527195
>равномерным вращением
Так направление же меняется, и сила постоянно возникает, которая натягивает трос или материал диска
И если бы его не было, воспринимающую эту силу материала, то оно бы по прямой продолжило двигаться, а не по кругу.
Аноним 23/01/21 Суб 17:10:28 52719785
>>527196
Ага, то есть точка на периферии замедляется относительно мгновенного направления поступательного движения. Спасибо.
Аноним 23/01/21 Суб 17:25:32 52719886
>>527196
Но все равно есть наркоманский вопрос: откуда эта сила знает
когда ей натягивать трос, т.е. откуда берется точка отсчета для вращения? Или как все как обычно: если сила не действует, то и вращения нет?
Аноним 23/01/21 Суб 18:01:45 52720087
>>527198
А у тебя точно не перепутана причина и следствие?
Аноним 23/01/21 Суб 19:19:00 52720388
Какую энергию производит один хомяк, когда он бежит в колесе? Сколько хомяков понадобится, чтобы, например, зарядить телефон?
Аноним 23/01/21 Суб 19:19:45 52720489
>>527200
Не знаю. Но если я что-то и путаю, то хотелось бы знать где.
Аноним 24/01/21 Вск 01:38:04 52722090
>>527204
Там где появляется какая-то магия в твоей фантазии.
Аноним 24/01/21 Вск 01:48:30 52722191
Есть тут шарящие в отражении?

Вот угол падения равен углу отражения
Но от чего зависит сколько отразится от прозрачного материала а сколько пройдёт через него?
От угла зависит?
От материала? Какой параметр это отражает?
Аноним 24/01/21 Вск 01:54:00 52722292
какой химический процесс осаждения углекислого газа в воде на дно океанов?
Аноним 24/01/21 Вск 03:19:23 52722493
>>527222
Чуть подкорректирую, где там кальций и почему в воде плавает, почему сам на дно не осождается а соединяется с углекислым газом?
Аноним 24/01/21 Вск 08:42:58 52723194
>>527179
> Поле есть? Есть. Притяжение есть? Есть. Вопросы?
Гравитация и электромагнетизм - абсолютно разные вещи, долбоеб. Гравитация даже не является фундаментальным взаимодействием, в отличие от электромагнетизма. Гравитация - это просто искривление пространства-времени, и это никакая не абстракция, а голый факт. Ты какой-то ебанутый.
Ядерные реакции Аноним 24/01/21 Вск 11:40:26 52724195
5bcd07f437acc23[...].png 525Кб, 741x719
741x719
Снова выхожу на связь. Вопрос такой: насколько целесообразно использовать термоядерный реактор для распада газообразного изотопа. Например углерода-14 или радона-220. Тоесть произвести не синтез, а распад с последующим выделением энергии. По моей идее сжатие и разогревание газообразного изотопа снизит критическую массу и начнёт распад газа.
Аноним 24/01/21 Вск 11:57:24 52724496
2222.png 17Кб, 1180x448
1180x448
Вообще моя идея в том чтобы сделать прямоточный ядерный ракетный двигатель, где делящаяся плазма сама и будет рабочим телом. Только главная задача в том чтобы максимально снизить критическую массу распада.
>>527241
Аноним 24/01/21 Вск 11:59:50 52724597
2222м.png 45Кб, 1106x417
1106x417
Аноним 24/01/21 Вск 12:01:47 52724698
Во нашел:
Критическая масса обратно пропорциональна квадрату плотности материала, что позволяет, например, при увеличении плотности вдвое, уменьшить критическую массу в четыре раза. Нужную степень подкритичности можно получить уплотнением делящегося материала за счет взрыва заряда обычного взрывчатого вещества, выполненного в виде сферической оболочки, окружающей ядерный заряд.
>>527241
Аноним 24/01/21 Вск 12:25:53 52724799
2222м.png 59Кб, 1432x498
1432x498
Во ёпт
Аноним 24/01/21 Вск 13:01:17 527248100
Хватит ли человечеству ресурсов на то, чтобы покрыть все пустыни на экваторе фолгой, чтобы увеличить отражение Солнца от Земли, понизить температуру, превратить всю планету в ледяной шар?
Аноним 24/01/21 Вск 13:41:29 527249101
>>527248
Не вижу последовательности в описанных тобой планах сумрачного гения. Как покрытие пары процентов суши фольгой повлияет на температуру Земли? У тебя три четверти суши уже зеркалом покрыты (океаном).
Аноним 24/01/21 Вск 13:50:53 527250102
>>527249
>зеркалом (океаном)
Хуясе ты гений
Аноним 24/01/21 Вск 13:55:07 527251103
>>527241
Как ты заебал уже со своей хуйней. Ты даже распад от деления отличить не можешь, еблан. Какой нахуй углерод-14? Ты вообще понимаешь, что использовать не чистые изотопы, а химические соединения делящихся изотопов - газ UF6 (гексафторид урана) например.

Только в сухом остатке это нахуй не нужно. Нахуя тебе сжимать газ, плотности металлического топлива ты не достигнешь всё равно.
КПД будет пиздец низкий. Меньше процента делящегося нуклида пойдёт на деление, остальное улетит в трубу.

В общем идея хуйня, рекомендую тебе хотя бы смешариков посмотреть перед тем как срать на доску.
Аноним 24/01/21 Вск 13:58:11 527252104
>>527249
Читал о том, что можно запустить цепную реакцию к наступлению вечной зимы. Например, Сахара, Аравия, Ближний Восток, Средняя Азия, Мексика производят огромное количество тепла. Их можно было бы постепенно покрывать фольгой, и тогда средняя температура на Земле начнёт падать. Зима станет дольше и будет охватывать больше площади, возобновится рост льдов в горах и на полюсах. Среднегодовое альбедо поверхности увеличится, и это снова приведёт к похолоданию, и так до тех пор, пока Земля полностью не покроется снежной шапкой.

Мне кажется, это вполне реальный осуществимый план того, как человечество может начать апокалипсис на Земле.
Аноним 24/01/21 Вск 14:23:52 527256105
>>527252
>как человечество может начать апокалипсис на Земле.
*но только для себя.
Аноним 24/01/21 Вск 14:29:42 527258106
>>527252
>к наступлению вечной зимы
Вулканическая активность продолжится, а поглощение водой углекислого газа станет практически 0 из-за льда везде, углекислый газ будет накапливаться, потеплеет и всё оттает.
Аноним 24/01/21 Вск 14:38:31 527261107
>>527258
Это займёт довольно много времени. Десятки, а то и сотни миллионов лет. "Вечно" именно относительно человеческой жизни.
Аноним 24/01/21 Вск 14:46:55 527262108
>>527261
Относительно человеческой жизни можно просто грязных бомб накидать на поверхность суши и всё. Природа будет как сейчас в Чернобыле, и никаких людей.
Аноним 24/01/21 Вск 14:49:28 527263109
>>527220
Какая магия-то? Вот смотри - ты запускаешь ракету с экватора на орбиту Земли. Если на восток запускаешь, то к твоей ракете уже прибавилось 1,5 км/с к скорости движения по орбите, если на запад - то столько же отнялось. Все вроде бы логично, но блядь... почему все это работает относительно "неподвижных звезд"? Почему у каждого тела не может быть своей точки отсчета?
Аноним 24/01/21 Вск 14:50:10 527264110
>>527262
Ядерный апокалипсис довольно легко переживается. У меня вообще есть теория, что мы, русские, станем победителями в ядерной войне. И последствия для земляшки даже от самой "грязной" ядерной войны не будут такими жёсткими. За пару десятков тысяч лет всё восстановится.
Аноним 24/01/21 Вск 15:05:25 527269111
>>527264
>У меня вообще есть теория, что мы, русские, станем победителями в ядерной войне.
Безусловно. Только наша русская кожа потемнеет, глаза станут уже, волосы чёрные и кучерявые, хуи станут обрезанными. Мы, русские, уже далеко не в первый раз так успешно побеждаем.
Аноним 24/01/21 Вск 15:05:48 527270112
>>527263
>почему все это работает относительно
Что не работает, как не работает?
Аноним 24/01/21 Вск 15:10:44 527273113
>>527251
>Ты даже распад от деления отличить не можешь
Деление это вид распада. Дальше что?
>газ UF6
Это тут вообще причём?
>Только в сухом остатке это нахуй не нужно. Нахуя тебе сжимать газ, плотности металлического топлива ты не достигнешь всё равно.
КПД будет пиздец низкий. Меньше процента делящегося нуклида пойдёт на деление, остальное улетит в трубу.
Ну так я и задаю вопросы потому что хочу уточнить.
>Меньше процента делящегося нуклида пойдёт на деление, остальное улетит в трубу.
Труба может быть очень узкой. Я сделал только примерный набросок. Я не имел ввиду какой либо конкретный изотоп. Я имел ввиду жидкий либо газообразный изотоп, который можно применить похожим на ядерное оружие либо ядерный реактор образом.
Аноним 24/01/21 Вск 15:12:22 527274114
>>527251
>Ты даже распад от деления отличить не можешь
Деление это вид распада. Дальше что?
>газ UF6
Это тут вообще причём?
>Только в сухом остатке это нахуй не нужно. Нахуя тебе сжимать газ, плотности металлического топлива ты не достигнешь всё равно.
>КПД будет пиздец низкий. Меньше процента делящегося нуклида пойдёт на деление, остальное улетит в трубу.
Ну так я и задаю вопросы потому что хочу уточнить.
>Меньше процента делящегося нуклида пойдёт на деление, остальное улетит в трубу.
Труба может быть очень узкой. Я сделал только примерный набросок. Я не имел ввиду какой либо конкретный изотоп. Я имел ввиду жидкий либо газообразный изотоп, который можно применить похожим на ядерное оружие либо ядерный реактор образом. Если критическая масса обратно пропорциональна плотности - значит надо подумать о плотности.
Аноним 24/01/21 Вск 15:16:27 527275115
>>527269
>русская кожа потемнеет, глаза станут уже, волосы чёрные и кучерявые, хуи станут обрезанными
Бред. Не будет никакой ядерной зимы и ослабления озонового слоя тоже.
Аноним 24/01/21 Вск 15:18:19 527276116
>>527270
>Что не работает, как не работает?
>Если на восток запускаешь, то к твоей ракете уже прибавилось 1,5 км/с к скорости движения по орбите, если на запад - то столько же отнялось.
Аноним 24/01/21 Вск 15:48:01 527278117
799px-GFJRD(Ori[...].jpg 38Кб, 799x317
799x317
ГФЯРДспрозрачно[...].JPG 127Кб, 1607x699
1607x699
>>527251
А вот и газофазный двигатель. Теоритически если энергию плотно направить на миллиграмм урана - этот уран вполне можно испарить и ионизировать. Надо же. И кто тут дурак?
https://traditio.wiki/%D0%93%D0%B0%D0%B7%D0%BE%D1%84%D0%B0%D0%B7%D0%BD%D1%8B%D0%B9_%D1%8F%D0%B4%D0%B5%D1%80%D0%BD%D1%8B%D0%B9_%D1%80%D0%B5%D0%B0%D0%BA%D1%82%D0%B8%D0%B2%D0%BD%D1%8B%D0%B9_%D0%B4%D0%B2%D0%B8%D0%B3%D0%B0%D1%82%D0%B5%D0%BB%D1%8C
Аноним 24/01/21 Вск 15:51:26 527279118
>>527251
Так что моя идея газофазного фузора - нихуя не глупость История ядерного газофазного двигателя начинает свой путь с конца 50-х годов, в то время когда человечеством была достигнута устойчивая технология производства ядерной энергии в реакторах и был накоплен солидный объём данных о ядерном топливе и его поведении в самых различных условиях эксплуатации. Сама возможность реализации принципов деления ядерного топлива в газообразной фазе предопределила и стремление учёных-атомщиков и разработку газообразного ТВЭЛа. Появилось несколько схем устройства газофазных ядерных реакторов и ракетных двигателей, но существенное отставание в практическом материаловедении от теоретических разработок не позволило создать рабочий образец газофазного реактора и ракетного двигателя по сегодняшний день. Трудности практической реализации оказались весьма велики, и в их ряду: организация устойчивого деления, создание критических условий в ГТВЭЛ (газофазный топливный элемент), подбор тугоплавких конструкционных материалов, и КПД деления топлива, оказались полностью взаимоувязаны и представляют комплексную проблему. В связи с этим разработчики стали вести поиски направлений реализации ГТВЭЛ в области физического удержания урановой плазмы с помощью сверхсильных магнитных полей. В этом направлении был достигнут наиболее ощутимый успех, и в этом прямая заслуга атомщиков-профессионалов России, но как и прежде ещё существует значительная масса технических трудностей. Общий ход и динамика разработки газофазного ядерного реактивного двигателя (равно и реактора) в настоящий момент позволяют утверждать что появление первых серийных и длительно работающих двигателей и реакторов большой мощности и с высокой степенью надёжности - перспектива ближайших 8-12 лет (2019-2029.г.г). Ведущей страной в направлении разработки и конструирования ГФЯРД является Россия.
Аноним 24/01/21 Вск 15:55:40 527280119
GasCoreopencycle.png 9Кб, 662x436
662x436
Аноним 24/01/21 Вск 16:04:01 527282120
>>527276
Шизик, ты же сам сказал что это как раз работает и понятно
Аноним 24/01/21 Вск 16:15:19 527283121
В скольки мерном мире мы живём? На данный момент сколько измерений принято считать учёными, 3, 4, больше? Длинна, ширина, высота, время, 4 получается?
Аноним 24/01/21 Вск 16:27:49 527284122
>>527283
>время
какое блять время...
Аноним 24/01/21 Вск 16:31:23 527285123
>>527283
>В скольки мерном мире мы живём?
ИРЛ все гипотетически точные величины кратны размерам и формах фундаментальных частиц, размеры и формы которых мы не знаем. А все наши системы измерений - это наш манямирок, ровно настолько точный насколько это нам нужно для конкретной ситуации. Так что измерений нет, либо их два - прямая и окружность.
Аноним 24/01/21 Вск 16:32:01 527286124
eaae3b08982eea3[...].png 157Кб, 775x535
775x535
И вот с хабры красота:
Аноним 24/01/21 Вск 16:51:08 527289125
Аноним 24/01/21 Вск 16:51:10 527290126
>>527283
>В скольки мерном мире мы живём?
Если ты имеешь в виду конкретно нас, то геном человека 0.7-мерный, а развёрнутый организм 2.7-мерный. При этом разные органы имеют разные мерности, например, сетчатка 1.7-мерная.
Аноним 24/01/21 Вск 16:51:54 527291127
>>527285
Я про конкретные величины не спрашивал.
Аноним 24/01/21 Вск 17:31:25 527295128
>>527273
>Деление это вид распада. Дальше что?
Сказочный долбоёб.
Спонтанное деление это вид распада.
Когда говорят о делении в реакторах, хуякторах, имеют ввиду вынужденное деление ядер.

>Это тут вообще причём?
При том, что ты не в курсе, что можно использовать делящиеся нуклиды, засунутые в разные химические элементы.

>>527278
>И кто тут дурак
Ты)
Я тебе уже отвечал максимально корректно про этот двигатель, но ты опять побежал срать в тред делящимся (sic!) углеродом

>>527283
В трёхмерном. Всё остальное - математические абстракции.
Аноним 24/01/21 Вск 17:38:14 527296129
>>527295
>В трёхмерном. Всё остальное - математические абстракции.
Обосрался с этого архитектора.
Аноним 24/01/21 Вск 17:47:02 527300130
>>527296
Ну тогда мы живём в бесконечно-мерном фазовом пространстве.

Сам посуди, движение каждой молекулы во вселенной описывается в, как минимум, шестимерном фазовом пространстве (Три координаты и три фазовые скорости)
А ведь в фазовом пространстве могут быть какие угодно величины: углы полёта, энергия, давление, время.

И теперь сам подумай, какая будет размерность фазового пространства всей вселенной, где неисчислимое количество молекул.
Аноним 24/01/21 Вск 18:52:17 527302131
>>527300
Если одно выражается через другое, то считай это минус размерность. Если так срезать размерности, то приходишь к голографическому принципу.

Теория напоминает платоновскую аллегорию пещеры и состоит из двух основных утверждений:

Вся информация, содержащаяся в некой области пространства, может быть представлена как «голограмма» — теория, которая «живёт» на границе этой области.
Теория на границах исследуемой области пространства должна содержать, самое большее, одну степень свободы на планковскую площадь.

Источником этой гипотезы служит так называемое AdS/CFT-соответствие, которое однозначно показывает математическую эквивалентность квантовой теории поля в пространстве-времени анти-де Ситтера размерности 3+1 и конформной теории поля в пространстве-времени размерности 2+1.
Аноним 24/01/21 Вск 19:09:20 527303132
>>527302
>Если одно выражается через другое, то считай это минус размерность. Если так срезать размерности, то приходишь к голографическому принципу.

Ну как у тебя например, связаны нейтрон в точке фазового пространства (x1,y1,z1,phi1,teta1,E1,t1) с нейтроном в точке фазового пространства (x2,y2,z2,phi2,teta2,E2,t2).

Ответ: никак, в общем случае никаких связей на микроскопическом уровне наложено быть не может.

Так что вопрос про размерность пространства бессодержателен: сколько надо для каких-то конкретных изысканий, столько и будет.

унеси это лженаучное говно на канал ударитьмусор
Аноним 24/01/21 Вск 20:35:02 527307133
Если оставить пустой термос на антарктиде на месяц, а потом замерить температуру точным градусником снаружи и внутри, то температура будет совпадать до тысячных градуса?
Аноним 24/01/21 Вск 20:57:31 527308134
>>527307
Нет, ты всё правильно понял, всегда будут какие остаточные следы
Только они станут такими слабыми очень быстро, что ты не сможешь увидеть разницы между всякой другой хернёй вносящей шумы различные
Аноним 24/01/21 Вск 21:09:02 527309135
Аноны, логическая задачка
Или геометрического анализа... хз

Короче есть источник света и есть приёмник
Какой формы должна быть коробка, чтобы идеально равномерно распределить всё излучения от источника света вне зависимости от его формы?
Он может расходиться на 180 градусов, а может быть сфокусированным и расходиться с углом 5 градусов
Аноним 24/01/21 Вск 21:11:09 527310136
maxresdefault.jpg 103Кб, 1280x720
1280x720
>>527291
Ну тогда все круглое и смазанное и измерений ровно столько сколько чисел после запятой в числе Пи, чтобы посчитать точно все длины и окружности
Аноним 24/01/21 Вск 21:14:08 527311137
T&GH-TomBombadil.jpg 34Кб, 260x300
260x300
Кто такой Том Бомбадилл?
Хочу именно ученым задать вопрос. К науке у меня доверие большое.
Аноним 24/01/21 Вск 21:23:16 527313138
>>527311
Даже учёные не такие задротики, это бан.
Аноним 24/01/21 Вск 21:23:57 527314139
>>527313
Ну а как же культурология и лингвистика? Это уже не науки чтоле?
Аноним 24/01/21 Вск 21:25:15 527315140
Аноним 24/01/21 Вск 21:27:58 527316141
>>527315
Но есть же наука, которая изучает художественную литературу?
Аноним 24/01/21 Вск 21:29:06 527317142
Аноним 24/01/21 Вск 22:02:51 527320143
>>527286
Это имеет смысл в космиечских двигателях, учитывая что сколько бы у тебя не было энергии, тебе придётся везти с собой хуйню которой придётся "стрелять", т.е. выкидывать для получения импульса
А так газ и выкидываем и он и топливо, частично.

Но проблема в том что у них очень маленькое время полураспада, по крайней мере у тех что ты раньше называл.
Приноси газообразную радиоактивную херню с периудом полураспада хотя бы 100 лет, тогда что-то такое мэйби покатит.

но 100 лет вопрос при какой плотности (давление x температура)
Но давай хотя при н.у. 100 лет полураспада найди такой газ.
Аноним 24/01/21 Вск 22:11:11 527321144
Так бляяя

Фотонный двинатель это самая эффективная хуйня или нет??
Раз 5 уже тут пробовал задавать этот вопрос, но каждый раз что-то невнятное.

У фотона же есть импульс по ме^2
Скорость "выбрасывания" у него как у "продукта сгорания топлива"-максимально возможная
Кпд светодиодов по энергетическому выходу уже около 80%
ДА БЛЯ, ЭТО Ж ПОЛУЧАЕТСЯ ТУПО ТЕПЛОВОЕ ИЗЛУЧЕНИЯ ТОЖЕ ФОТОНЫ
Выходит свет можно излучать с кпд от затраченной энергии почти 100%.

Эффективность ракетного двигателя зависит от скорости истекания топлива, забыл как этот параметр называется.. удельный импульс вроде

Так что, фотонные двигатели тупо на светодиодах это топ или не топ?

И вот что ещё, можно же солнечными панелями ловить свет от звезды, получается оно ловит их, импульс поглотила панель, полетел корабль сильнее, далее ток выработала, на саетодиод подал и опять-таки полетела быстрее
кпд 200% ебать
(хотя вангую импульс от отражённого фотона будет равен импульсу от словленного солнечной панелью и испущенного светолиодом, если кпд этого процесса будет условно 100%)
Аноним 24/01/21 Вск 22:16:17 527322145
>>527320
Ну да есть такое. Я не силен в ядерной физике. Мне кажется это может быть как природный изотоп, так и крафтовый из ядерных реакторов либо же даже соли классических изотопов. Но опять же упираемся в материаловедение ибо надо очень сильно сжать эти соли чтобы снизить массу до приемлемой и управляемой величины. Мне стали интересны как изотопы благородных газов так и торий и полоний.
Аноним 24/01/21 Вск 22:18:36 527324146
>>526419 (OP)
Анон теор-физик, ты тут?
Я думаю о поступлении на физику. Как там происхдит обучение? Много ли лабораторных? Всякие измерения, опыты и тд?
Аноним 24/01/21 Вск 22:58:13 527328147
>>527303
>унеси это лженаучное говно
Ты ебанутый? Эту хуйню придумали, пока решали проблему энтропии черных дыр.
>никак, в общем случае никаких связей на микроскопическом уровне наложено быть не может.
Подожди... фи и тетта это у тебя что прости?
Аноним 24/01/21 Вск 23:23:23 527330148
15735742454670.jpg 278Кб, 800x800
800x800
>>526419 (OP)
Почему ёмкость конденсатора зависит отрасстояния между обкладками? Как это работает? Вот на одной обкладке избыток электронов, а на другой недостаток. Причём здесь расстояние?
Аноним 24/01/21 Вск 23:27:18 527331149
>>527330
Напряжение ты куда дел? Знаешь как напряжение связано с электрическим полем?
Аноним 24/01/21 Вск 23:34:11 527332150
>>527324
А в чём вопрос? Не хочешь много делать уроков?
Аноним 24/01/21 Вск 23:35:16 527333151
>>527330
Через силу-меньше расстояни, больше сила притяжения/отталкивания между разными зарядами плюсовыми или минусовыми.
Аноним 24/01/21 Вск 23:36:44 527334152
А вот может кто объяснит почему именно через фарады ёмкость конденсаторов измеряют, а не через обычные амперчасы?
Чем фарады удобнее?
Аноним 24/01/21 Вск 23:43:51 527335153
>>527331
Напряжение - это разница между зарядами. Не понимаю о чём ты.

>>527333
А как это связано со способностью конденсатора удерживать заряды?
Вот допустим мы наэлектризовали диск. Вкачали в него кучу электронов и оставили его летать на орбите в вакууме. Ну или закутанный в толстый слой диэлектрика. В таком диске заряд будет хуже держаться, чем аналогичном диске, но рядом с которым находится диск с дефицитом электронов.
И я пытаюсь понять физический смысл этого феномена, как это работает. Интересно же.
Аноним 24/01/21 Вск 23:59:28 527337154
>>527335
>А как это связано со способностью конденсатора удерживать заряды?
Так а куда они денутся?
В реальном конденсаторе понятно есть утечки и довольно быстрый саморазряд
Но в идеальный конденсатор теоретический если рассматривать-закачали с помощью напряжения источника которое больше чем то что в конденсаторе в него электроны/протоны
Убрали напряжение, и куда они оттуда денутся эти частицы?
Аноним 25/01/21 Пнд 00:00:23 527338155
Пацаны, читаю у Лема такую фразу "... вездеходы оставляли за собой клубы пыли", действие происходит на Луне. Будет ли пыль клубиться на Луне?
Аноним 25/01/21 Пнд 00:00:30 527339156
>>527337
Два диска на близком расстоянии удерживают больше чем те же два диска на большом.
Почему?
Аноним 25/01/21 Пнд 00:08:37 527340157
>>527339
Потому что ты неправильно ответил на вопрос о напряжении, поэтому ты не можешь понять почему. Потому что ответ в энергии электрического поля.
Аноним 25/01/21 Пнд 00:30:20 527342158
>>527339
Они ужерживают не больше частиц, а между частицами больше СИЛА, т.к. меньше расстояние, т.к. электрическое взаимодействие ослабляется обратноквадратично или как-то так.
Аноним 25/01/21 Пнд 00:33:29 527344159
>>527338
Нет конечно, ну как, будет опадать с ускорением свободного падение на луне, любого размера пыль, даже очень мелкая, которая у нас летала бы часами
Но там ускорение свободного падения очень маленькое, наверное было бы в каком-то роде похоже на клубление пыли.
Но вопрос как её подбросить, на земле же пыль воздух обычно поднимает, а не сами колёса, ветер от корпуса машины
А там только если бы протектор высокий на покрышке загребал её и подбрасывал

Короче возможно, но очень не так как тут
Аноним 25/01/21 Пнд 00:36:03 527345160
Screenshot2021-[...].png 64Кб, 720x1280
720x1280
>>527309
Это типа не решение, а пояснение к вопросу, источники света с разным углом расхождения
Аноним 25/01/21 Пнд 00:37:49 527346161
>>527340
Да. Признаю что не понимаю. Потому и прошу объяснить.

>>527342
Ты уверен? Может заряд - это всёже только электроны или их недостаток?
Аноним 25/01/21 Пнд 00:42:53 527349162
>>527346
А то в чём измеряются конденасторы это заряд?
Такой же в котором измеряется заряд элементарных частиц?
Аноним 25/01/21 Пнд 00:59:03 527352163
Аноним 25/01/21 Пнд 01:34:40 527355164
>>527352
Конденсаторы же в фарадах а у электрона кулон
Я не разбирался, почитай определения того и того.
Аноним 25/01/21 Пнд 01:46:03 527358165
>>527324
>Я думаю о поступлении на физику
Пока ты не скажешь куда именно, твой вопрос не содержателен.
Если в рашке на бакалавра, то лабы у тебя будут обязательно, вопрос о качестве. На физтехе лабы по общей физике полный кал, слава богу их всего 6 семестпов но у кафедры может быть содержателтный доп курс. Вышка хз, у нас бегает один преп, говорит, что у них пытаются сделать лабы "на подумать" вместо тупого следования методички, в успехе сомневаюсь, но это уже не ко мне.За бугром у теорфизиков лаб может и не быть - там образование не фундаментальное, а прикладное, и тебе вместо лаб просто курс по теории представлений/категорий/другой теорсосной матеше пихнут.
Аноним 25/01/21 Пнд 01:51:57 527360166
psimple2d.mov 8373Кб, 448x432, 00:00:37
448x432
Газодинамики тут есть?
Есть линеаризованный метод годунова с двумерной сеткой.
Когда считаем большие импульсы вдоль осей, как посчитать импульс перенесенный от боковой (параллельной оси) стенки? Нужно вводить вязкость? Я заебался видеть эти кривые волны халп
давление видеорил и это не похоже на воздух
хотя сумарная энергия после 100к итераций не изменилась ни на бит
Аноним 25/01/21 Пнд 01:52:16 527361167
>>527334
Фарад это не просто заряд конденсатора, а заряд конденсатора, напряжение между обкладками которого равен 1 вольту, в этом и разница.
Аноним 25/01/21 Пнд 02:03:31 527362168
>>527345
гиперболоид
спутниковые тарелки видел?
Аноним 25/01/21 Пнд 02:07:51 527363169
>>527362
Вроде же он должен в одну точко собрать "рассеяный" сигнал, а мне наоборот надо рассеять, причём одинаково что сфокусиорванный луч, что и так уже рассеяный.
Аноним 25/01/21 Пнд 02:08:38 527364170
>>527361
И тебе этого не хватило для понимания?
Аноним 25/01/21 Пнд 02:08:58 527365171
>>527346
Значит смотри. Есть три величины, которые не стоит путать. Первая, это электрический потенциал (короче фи которая) именно её изменение называют напряжением (это вторая штука). Если в одной точке фи 100 а в другой фи 300 то между этими точками 200 вольт напряжения.
Третья характеристика, это короче электрическое поле, которое обозначают вектором E. Этот вектор, это короче просто градиент от электрического потенциала. Если тебе нужно найти напряжение между точками A и B то ты либо просто вычитаешь из элетрического потенциала в точке B электрический потенциал в точке А, либо интегрируешь траекторию из A в B с помощью электрического поля (так как оно градиент, то это одинаковая хуйня даже с точки зрения математики).

А теперь смотри, у тебя есть плоский конденсатор (будем считать идеальным). По довольно простому расчету через уравнение Максвела, выходит, что между обкладками такого конденсатора, электрическое поле это тупо одинаковые вектора от одной обкладки до другой, причем длина вектора E просто пропорциональна заряду на обкладке. Это значит (раз это производная от потенциала) что потенциал от одной обкладки до другой тупо линейно растет. Теперь берем два конденсатора, один толще другого в два раза. Подключаем их к одному и тому же напряжению. Что получаем, так как напряжение U=EL (банальное интегрирование, дает такой результат), то увеличивая L, уменьшается E, а раз уменьшается Е, значит уменьшается заряд на обкладках, а значит и на обкладке заряда нужно меньше.
Аноним 25/01/21 Пнд 02:14:12 527368172
>>527363
если источник в фокусе будет, то вообще не важен угол расхождения - все лучи после отражения будут параллельны оси ПАРАБОЛОИДА
я ошибся нужен параболоид а не гиперболоид)
Аноним 25/01/21 Пнд 02:20:19 527370173
>>527363
и да если тебе надо собрать потом весь свет в приемник то можно ззаюзать эллипс: приемник и источник в фокусы. Тогда как бы ты не рассеивал свет он весь попадет на приемник
гугли овальный бильярдный стол от намберфайл тебе зайдет
Аноним 25/01/21 Пнд 02:20:53 527371174
>>527368
Так это если он будет идеально зеркальным получается?

Я просто думал о матовой поверхности
Матовую с зорошим отражением полного спектра сделать гораздо легче чем зеркальную
Хотя... Идея хорошая, можно по крайней мере первично отражать/рассеивать параболоидом, а затем матовой штукой уже как надо...
Аноним 25/01/21 Пнд 02:25:05 527373175
>>527370
>приемник и источник в фокусы.
А вот это на магию похоже

При разном угле расхождения луча будет в фокусе и источник и приёмник, за 1 отражение?
Просто если оно в случае зеркальной поверхности будет много раз летать туда-сюда отражаться то это не вариант, слишком много потерь будет и неправильные данные.

Нет, собирать весь свет в датчик не нужно, нужно чтобы не зависимо от угла расхождения источника, формировалась равномерная область засветки.

Например 2 источника ОДИНАКОВОЙ МОЩНОСТИ световой, но с разным углом расхождения пучка, должны на датчике дать в итоге ошинаковое значение.
Аноним 25/01/21 Пнд 02:39:08 527376176
>>527373
да за одно отражение из фокуса в фокус - это геометрия
еще раз говорю - глянь на овальный бильярдный стол и все поймешь

как сделать зеркальный параболоид в домашних условиях:
берешь оловосвинец в банку, плавишь, раскручиваешь банку, не останавливая вращения даешь застыть, паста гои + вата + дрель
чем быстрее крутишь тем меньше парабола получится
чем меньше парабола - тем меньше разница в ширине пучка при разных углах расхождения
Аноним 25/01/21 Пнд 02:41:10 527378177
>>527328
>Ты ебанутый? Эту хуйню придумали, пока решали проблему энтропии черных дыр.
Докен доказал, кстати.
>Подожди... фи и тетта это у тебя что прости?
Азимутальный и полярный угол. Хотя это без разницы, можно назвать обобщенной координатой.

>>527322
Господи, блядь. Прошу, нахуй, перед тем как хоть что то писать в тред почитай, что такое:

(n,f) реакция
Сечение ядерной реакции

Потом открой JANIS и убедись, что самые распространённые изотопы полония, тория и прочего говна которое ты тут пишешь нихуя не делятся нормально. Ты путаешь тёплое с мягким: активность элемента и его склонность к делению.
Аноним 25/01/21 Пнд 02:41:40 527379178
>>527376
>как сделать зеркальный параболоид в домашних условиях:
Не, там надо полнота отражаемого спектра и прочая хуета...
Хотя может олово и даёт такое, хотя вряд ли
Так-то вроде только хром в этом нормальный
Аноним 25/01/21 Пнд 02:57:50 527384179
>>527358
Зарубеж офк, выбираю из топовых уников.
Мне интересно как само обучение устроено. В школе например, физика это куча лабораторных, чуть ли не половина занятий.
Аноним 25/01/21 Пнд 02:59:31 527386180
>>527355
А причём здесь это?
Аноним 25/01/21 Пнд 03:07:01 527387181
>>527386
Ну он же не понимал как конденсатор становится "более ёмким" если уменьшить расстояние между обкладками.
Он становится более ёмким по энергии, а не по количеству частиц.
Аноним 25/01/21 Пнд 03:07:03 527388182
>>527365
Это объяснение выведенное из формул. А я не могу понять физический смысл. Что мешает лишним электронам из отрицательно зараженной пластины вылетать, когда рядом находится положительно заряженная пластина, и почему когда пластины нет, они вылетают?
Электроны такие звонят электрону-диспетчеру, а он по формуле считает, пора ли им улетать, и даёт команду когда по формуле расстояние станет подходящим?
Или разницы Нет? Тогда почему конденсатор состоит из двух пластин? Можно было бы взять одну пластину, обмотать диэлектриком, вкачать туда электронов, и получился бы накопитель энергии аналогичный конденсатору.
Аноним 25/01/21 Пнд 03:08:57 527389183
>>527388
>электронам из отрицательно зараженной пластины вылетать, когда рядом находится положительно заряженная пластина, и почему когда пластины нет, они вылетают?
Кто куда и почему вылетать должен?
Аноним 25/01/21 Пнд 03:21:53 527392184
>>527378
>Азимутальный и полярный угол. Хотя это без разницы, можно назвать обобщенной координатой.
Они же блять выражаются через декартовы. А значит их можно выкинуть нахуй.
Аноним 25/01/21 Пнд 03:26:17 527394185
>>527388
>Тогда почему конденсатор состоит из двух пластин? Можно было бы взять одну пластину, обмотать диэлектриком, вкачать туда электронов, и получился бы накопитель энергии аналогичный конденсатору.
Да, такой конденсатор тоже есть. Но он просто хуже по емкости. Ты нового не придумал. А по поводу почему электроны не улетают, ну вообще-то улетают нахуй, после достижения определенного критического напряжения, конденсатор либо пробивает. Либо если он в вакууме, он превращается в рентгеновскую трубу. И от вида конденсатора и количества платин это не зависит, только от напряжения и материала конденсатора.
Аноним 25/01/21 Пнд 03:32:47 527395186
>>527388
>А я не могу понять физический смысл
Смысла физического не существует, найдешь пиши, а лучше звони, но не сюда а в Кащенко.

>Электроны такие звонят электрону-диспетчеру, а он по формуле считает, пора ли им улетать, и даёт команду когда по формуле расстояние станет подходящим?

У тебя какой-то странный подход. Есть законы физики. А есть объекты, которые этим законам подчиняются. Зачем ты включаешь сюда некую "свободу воли"? Анон алло, попал в черную дыру, всё каюк тебе. Попал в солнечную систему близко к солнцу, крутись по орбите. Всё описано и объяснено уже давно. Какой смысол ты ищешь? Это описание и фиксация. Нет там третьей величины, задающей смысл.
Аноним 25/01/21 Пнд 03:36:10 527396187
>>527388
>А я не могу понять физический смысл
Тип если ты ищешь ответ на вопрос:

Апочему электронам бы не трахнуть кварки, ну и похуй что педофилия. Алло. Педофилии нет в квантах. И в атомах. И в молекулах. Смысла нет в физиках. Есть законы. Они про то, как все работает. И всё. Это работает так, это работает сяк. Если это работает так-сяк то тебе или в Россию (гуманитарная шутка) или в квантовую физику
Аноним 25/01/21 Пнд 04:56:07 527402188
>>527394
> Либо если он в вакууме, он превращается в рентгеновскую трубу.
Как это?


>>527395
>Смысла физического не существует, найдешь пиши, а лучше звони, но не сюда а в Кащенко.
То что ты не понимаешь, не может понимать больше никто? Потому что ты считаешь себя самым умным? И вот такой человек говорит про кащенко. Хотя. если подумать, это и не удивительно.

>>527396
Ты похоже инвалид в плане этой способности, ты это понимаешь и у тебя от этого печёт. Тебе не позавидуешь.
Аноним 25/01/21 Пнд 04:58:06 527403189
>>527394
>Да, такой конденсатор тоже есть. Но он просто хуже по емкости.
Ровно в два раза чем у конденсатора с двумя обкладками?
Думаю нет, не в два.
Аноним 25/01/21 Пнд 06:20:29 527407190
>>527402
>Как это?
ЭЛЕКТРОНЫ ВЫЛЕТАЮТ И ЛЕТЯТ ИХ ВЫДАВИЛИ
Аноним 25/01/21 Пнд 07:12:30 527408191
>>527407
Получается бета а не рентген лучи.
Аноним 25/01/21 Пнд 07:50:22 527410192
>>527408
А я и не называл это рентгеновским излучателем
Аноним 25/01/21 Пнд 08:28:55 527411193
>>527410
Ты называл это рентгеновской трубкой. Это другое?
Аноним 25/01/21 Пнд 09:37:20 527415194
Вопрос физикам. Как можно разогнать нейтроны в ускорители частиц?
Аноним 25/01/21 Пнд 09:40:34 527416195
>>527408
Ускоряющиеся электроны хуярят рентгеновскими лучами... Они же излучать начинают.
Аноним 25/01/21 Пнд 09:41:01 527417196
>>527403
>Ровно в два раза чем у конденсатора с двумя обкладками?
Посчитай
Аноним 25/01/21 Пнд 10:33:17 527423197
>>527165
Я не про спектроскопию спрашивал.
Сейчас моден так называемый СВЧ катализ.
Это когда происходит активация катализатора с помощью СВЧ, а не с помощью термического нагрева печкой.
В том числе нагрева, например, раствора прекурсоров для синтеза, которые после нагрева в СВЧ дают те же результаты, что и при нагреве печкой, но гораздо быстрее.
Мой вопрос касается непосредственно изучения спектра поглощения веществ во всем диапазоне частот СВЧ, который очень широк на самом деле и начинается от сотен МГц и заканчивается уже к дальнего ИК.
Я видел крайне мало работ, где бы взяли, например, спирт сложный и показали его спектр поглощения во ВСЕМ диапазоне СВЧ. Или сравнили его с его же растворами.
Просто с современными возможностями генерировать СВЧ это вроде как не представляется тяжелой задачей.
Или это просто никому не нужно, потому что это уж совсем фундаментальная наука, которая денег не приносит?
Аноним 25/01/21 Пнд 12:09:09 527425198
>>527415
>Вопрос физикам. Как можно разогнать нейтроны в ускорители частиц?
Никак. Это нейтральная частица, а значит электромагнитным полем ее не разогнать.
Аноним 25/01/21 Пнд 12:13:07 527426199
>>527425
А если направить пучок нейтронов по касательной к массивному телу, то благодаря эффекту слингшота их скорость может возрасти? Ведь массой то они обладают?
Аноним 25/01/21 Пнд 12:43:37 527428200
>>527425
а если разогнать протон и всяким вашим квантовым кунг-фу превратить его в нейтрон?
Аноним 25/01/21 Пнд 13:08:40 527429201
>>527384
тогда пиздуй в сцай форча, там хотя бы есть много забугорных студентов, а то насоветуют тебе тут мамкины теоретики
Аноним 25/01/21 Пнд 13:48:06 527431202
>>527428
>протон и всяким вашим квантовым кунг-фу превратить его в нейтрон?
Не получится. Как ты представляешь себе такую магию?
Аноним 25/01/21 Пнд 16:13:55 527445203
>>527431
Прям как в нейтронных звёздах происходит.
Аноним 25/01/21 Пнд 16:16:30 527446204
Мне вот что интересно, возмущения во многих полях легко переходят/превращаются в возмущения в других полях
Хули высрали это типа фундаментальное якобы свойство сохранения электрического заряда?

Сначала просто высрали, на волне фантазии о невозможности "трансмутаций" а потом оно просто чисто случайно подтвердилось наблюдательно?
Аноним 25/01/21 Пнд 16:17:44 527448205
>>527411
Я про то что это был не я))

Кстати вроде там и излучение жёсткое хуярит, но я не уверен.
Аноним 25/01/21 Пнд 16:17:50 527449206
>>527445
А они излучают нейтроны?
Аноним 25/01/21 Пнд 16:19:17 527450207
>>527449
Я про превращение каких-то других херей в нейтроны

И да, наверное вполне хуярят, как гейзерами, когда поверхностная кора разламывается на нейтронных звёздах
Аноним 25/01/21 Пнд 16:33:17 527452208
>>527450
>Я про превращение каких-то других херей в нейтроны
Вроде нет такого превращения. Или есть?
Аноним 25/01/21 Пнд 16:34:54 527453209
>>527450
>нейтронных звёздах
Период полураспада нейтрона 10 минут.
Аноним 25/01/21 Пнд 16:46:41 527454210
>>527452
Хз, вроде про нейтроны говорили, когда смотрел про образование нейтронных звёзд
Там давлением вжимает что-то во что-то и получаются нейтроны.
вроде электроны в протоны.
Аноним 25/01/21 Пнд 16:47:00 527455211
Аноним 25/01/21 Пнд 16:58:28 527456212
>>527455
Свободного т.е. вне ядра. В нейтронных звездах нейтроны типа как свободные, вне ядер.
Аноним 25/01/21 Пнд 17:01:09 527457213
>>527456
Ну а насчёт давлений что?

В любом случае даже если распадаются то сразу же обратно нейтронами становятся
Аноним 25/01/21 Пнд 17:09:34 527458214
>>527457
А какого цвета нейтронная звезда?
Аноним 25/01/21 Пнд 17:11:02 527460215
Списку со случайным порядком из 10 букв ставится в соответствие 11 цифры ( сочетание типа а1, b2,c11 итд)
какова вероятность того, что в результате двух опытов выпадет одинаковое сочетание?
Аноним 25/01/21 Пнд 17:14:45 527462216
Аноним 25/01/21 Пнд 17:14:57 527463217
>>527429
Спасибо, напишу на форч.
Я думаю учёба везде +- одинковая, вне зависимости от страны.
Например, на матфаке слушаешь лекции, потом решаешь задачки. Экзамен это решение задач и док-во теорем.
Как на физики это происходит? Лабораторные, опыты и тд? Или этого нет и так же слушаешь лекции, потом задачи/отвечаешь на вопросы.
Аноним 25/01/21 Пнд 17:24:24 527464218
Аноним 25/01/21 Пнд 18:44:01 527469219
>>527457
Давление не влияет. (если конечно не убербольшое, где нейтроны слипаются)
Аноним 25/01/21 Пнд 19:09:28 527470220
>>527431
Погугли, что такое нейтронный генератор.

Можно облучать дейтронами ядра трития/дейтерия. Мишень превратится в гелий и из неё вылетит нейтрон, энергия которого будет пропорциональная энергии дейтрона.
Аноним 25/01/21 Пнд 19:21:37 527472221
>>527463
Братан, первые два курса для всех физиков и около-физиков одинаковые. Будешь сначала изучать матан, линал, урматы, общую физику, на которой и будут лабы.

Только со второй половины второго курса начинается терфиз: сначала термех, потом уже терпол, кванты, статы, рел. кванты, физкин и так далее.
Аноним 25/01/21 Пнд 22:07:29 527478222
>>527472
Спасибо, такой ответ я и ждал.
Аноним 26/01/21 Втр 04:49:48 527491223
Я правильно понял, что когда гамма-квант рассеивается на ядре водорода, то происходит рождение пары электрон-позитрон и последующая их аннигиляция с выделением нескольких гамма-квантов? Рассеивание идет на любом атманом ядре? На протии оно возможно?
Аноним 26/01/21 Втр 08:50:56 527500224
>>527491
Для рождение пар нужно чтоб гамма квант проходил через поле с очень высокой напряженностью. Протон в принципе может создать такую напряженность, но это мало вероятный процесс. Обычно этот процесс на ядрах с зарядом числом.
Аноним 26/01/21 Втр 09:27:14 527501225
>>527500
>Протон в принципе может создать такую напряженность
Как же тогда наблюдали рождения пар?
>поле с очень высокой напряженностью.
Напряженность поля равна E=kq/r^2 где k - коэффициент пропорциональности 9⋅109 Н*м2/Кл2, q - заряд, r - расстояние.
Заряд протона 1,6⋅10-19 Кл.
Заряд ядра кислорода 1,3⋅10-18 Кл.
Требуемая для рождения пар напряжённости поля E =1016 В/см
Сможешь рассчитать на каком расстоянии от ядра водорода и от ядра кислорода будут рождаться пары?
Аноним 26/01/21 Втр 14:12:54 527508226
>>527452
плюс бета распад
Аноним 26/01/21 Втр 15:33:13 527513227
>>527452
Находясь в достаточно глубокой потенциальной(любой природы, даже в неинерциальной системе отсчета) яме, протон превращается в нейтрон.
Вообще это касается всех нестабильных частиц и квазичастиц.
Аноним 26/01/21 Втр 15:37:11 527514228
>>527513
>протон превращается в нейтрон
И кто-то наблюдал как это происходит? Т.е. откуда ты это узнал?
Аноним 26/01/21 Втр 15:45:04 527516229
>>527514
Ну тащемта этот эффект теоретически предсказан еще в первых теориях ядра.
Собственно потенциальная яма, создаваемая взаимодействием Юкавы, позволяет нейтронам не распадаться в ядрах.
Аноним 26/01/21 Втр 16:18:29 527517230
>>527514
За образованием нейтронных звёзд астрономы наблюдали, и наверняка в коллайдерах всяких стреляли электронами по нейтронам.
Аноним 26/01/21 Втр 16:37:33 527520231
>>527516
>Ну тащемта этот эффект теоретически предсказан еще в первых теориях ядра.
Это было давно. А экспериментально это наблюдалось?
Аноним 26/01/21 Втр 16:41:14 527521232
>>527517
>За образованием нейтронных звёзд астрономы наблюдали
А в них происходит это?

>>527513
>Находясь в достаточно глубокой потенциальной(любой природы, даже в неинерциальной системе отсчета) яме, протон превращается в нейтрон
Аноним 26/01/21 Втр 17:11:49 527522233
>>527521
>А в них происходит это?
Образование нейтронов? да, конечно, иначе бы нейтронные звёзды ПО МОДЕЛЯМ были бы не такие какие есть сейчас
Вообще не происходило бы превращения из белых карликов в нейтронные звёзды
Аноним 26/01/21 Втр 17:15:17 527523234
>>527520
Вообще-то на этом основаны термоядерные реакции и не только.
Эффект косвенно наблюдается на рассеивание на ядрах и столкновение адронов.
Аноним 27/01/21 Срд 03:35:02 527539235
Аноним 27/01/21 Срд 04:42:47 527548236
АВТОМАТИЗИЯ ПРОИЗВОДСТВА
@
ОЖИДАНИЕ 100 ПРОГРАММИСТОВ И ИНЖИНЕРОВ
@
РЕАЛЬНОСТЬ: НЕМНОГО ЛУЧШЕ ЧЕМ В СССР

Почему нет почти пустых заводов и фабрик?
Почти без людей
Аноним 27/01/21 Срд 09:43:34 527558237
>>527548
>РЕАЛЬНОСТЬ: НЕМНОГО ЛУЧШЕ ЧЕМ В СССР
>Почему нет почти пустых заводов и фабрик?

В СССР идея была в том, что "автоматизируем"+за счет этого сокращаем рабочий день+деньги те же, но у всех больше времени. Но там до автоматизации всего далеко оч было.

А что теперь? А теперь автоматизируем всё, вышвыриваем толпы негров на улицы, и чего потом с ними делать? Платить им бабки никто не будет. Плюс к тому, далеко не всё можно автоматизировать даже сегодня. Да и дорого это в конце-концов. В долгосрочной перспективе конечно окупается, но разовых инвестиций кабы много надо. Я уже даже не буду затирать про другие аспекты этой тотальной автоматизации.

Это буквально история про охуенного программиста который всё автоматизировал в конторе, да так, что его уволили потому что не нужен, и перевели администрирование на аутсорс лол.

>Почти без людей
Есть же автоматические сборочные линии.
Аноним 27/01/21 Срд 09:46:23 527559238
>>527548
>Почему нет почти пустых заводов и фабрик?
>Почти без людей

https://www.theverge.com/2019/11/13/20962688/adidas-robotic-speedfactories-ansbach-germany-atlanta-usa-athletic-shoes

Рыночек порешал.
Незачем плодить сущности, если вьетнамец за миску риса выдает чуть меньше робота, которому требуется дорогостоящее обслуживание.
ЭВОЛЮЦИЯ Аноним 27/01/21 Срд 10:03:52 527562239
QFZaBmGn61k.jpg 54Кб, 427x632
427x632
Порекомендуйте хороших книг по теории эволюции, любого уровня
Аноним 27/01/21 Срд 11:16:16 527563240
>>527559
>если вьетнамец за миску риса
Вообще-то МРОТ в Камбодже, например, $192 в месяц т.е. больше чем в России. про Вьетнам не знаю, но думаю там МРОТ еще выше, чем в Камбо.
Аноним 27/01/21 Срд 11:23:42 527564241
Аноним 27/01/21 Срд 12:05:16 527565242
>>527563
МРОТ - это рекомендательное значение.
Аноним 27/01/21 Срд 13:03:12 527567243
>>527563
Мротами меряются только граждане одной соседней страны, это мало что значит вообще.
Аноним 27/01/21 Срд 14:35:38 527568244
>>527565
Разве? В России можно заключить контракт с работником и платит ему мене МРОТа?
Аноним 27/01/21 Срд 15:13:25 527571245
>>527568
Называется дробная ставка.
Аноним 27/01/21 Срд 16:06:15 527573246
>>527567
Это значит сколько платят пидорахам
Аноним 27/01/21 Срд 16:10:05 527574247
>>527559
Роботы могут делать роботов и потенциально обслуживать роботов, но для роботов нужен металл, а он дорогой, а въетнамец получается из углерода, т.е. из воздуха.
Тред тупых вопросов 27/01/21 Срд 21:44:53 527588248
>>527079
Пруфы будут? Показывают как пиздятся кулаками и кусают
Рыбников прав! Аноним 27/01/21 Срд 21:57:51 527591249
schet-drevnih-s[...].jpg 53Кб, 624x433
624x433
Рыбников прав! Не надо путать катоды и катоды с плюсами и минусами. Электродвижущая сила это разность потенциалов между двумя положительными (sic!), которая стремиться вернуть себе равновесие. Анологией здесь привести весы. Когда весы уравновешены - между ними ноль разницы. В момент когда на одну чашу весов положили предмет - между чашами появилась разница потенциалов. Но в чём конкретно прав Юрий Степаныч? А в том что положительный и отрицательный заряд это ситуативная абстракция, которая работает только в условиях задачи, но дураки-академики перенесли эту ситуативную условность на представления о фундаментальных взаимодействиях.
Аноним 27/01/21 Срд 21:58:13 527592250
>>527591
>катоды и катоды
катоды и аноды
фикс
Рыбников прав! Аноним 27/01/21 Срд 22:02:01 527593251
schet-drevnih-s[...].jpg 53Кб, 624x433
624x433
Рыбников прав! Не надо путать катоды и аноды с плюсами и минусами. Электродвижущая сила это разность потенциалов между двумя положительными (sic!) зарядами, которая стремиться вернуть себе равновесие. Анологией здесь привести весы. Когда весы уравновешены - между ними ноль разницы. В момент когда на одну чашу весов положили предмет - между чашами появилась разница потенциалов. Но в чём конкретно прав Юрий Степаныч? А в том что положительный и отрицательный заряд это ситуативная абстракция, которая работает только в условиях задачи, но дураки-академики перенесли эту ситуативную условность на представления о фундаментальных взаимодействиях.
и ещё фикс
Аноним 28/01/21 Чтв 06:05:22 527621252
>>527593
Кто приведёт конраргумент почему + это + а - это -?
А, я вспомнил о чём подумал-притяжение и отталкивание не относительно, если бы было просто + всё бы отталкивалось, но просто или слабее или сильнее, а так оно или отталкивается ++ -- или притягивается + -
Аноним 28/01/21 Чтв 07:20:33 527623253
>>527593
>Не надо путать катоды и аноды с плюсами и минусами.
Да, не надо.
ACID - Anode Current Into Device.
В остальном у тебя бред.
Аноним 28/01/21 Чтв 11:43:48 527627254
>>527621
Я могу привести аргумент. Знаешь какой?

Так договорились, но всегда, когда обсуждают заряды в теорфизе, добавляют, что выбор + и - произволен. Как хочешь так и выбирай, это не меняет физику никак.
Аноним 28/01/21 Чтв 13:30:47 527632255
Если людей обучать с детства самоконтролю, то получатся ли сверхлюди? Почему такая охуенная идея еще не реализована в мире? Или, может, реализована, но я про нее не знаю?
Аноним 28/01/21 Чтв 18:46:15 527644256
Как даются названия методам, уравнениям и т.п. в честь учёных? Вот жил-был Хуезадрищеский, придумал он формулу и написал где-то о ней. Само по себе это не сделало эту формулу Хуезадрищеской, порой ведь не в честь открывателя формулы её называют, или дают двойное имя и т.д. Но больше открывателей не нашлось, потому через пару веков будет формула Хуезадрищеского. Но когда её так начнут называть и примут это наименование?
Аноним 28/01/21 Чтв 20:00:42 527648257
index.jpg 7Кб, 282x179
282x179
>>527621
Притяжение это восстановление равновесия при недостатке, отталкивание же наоборот это восстановление равновесия при избытке. В любом случае + (плюс) это больше чем равновесие в конкретном случае периодической таблицы, а - (минус) это меньше чем равновесие. Ион это неуравновешенное ядро.
Аноним 28/01/21 Чтв 20:32:27 527650258
вы че тут устроили?
Аноним 28/01/21 Чтв 20:52:24 527652259
>>527650
Кафедру счёта древних русов
Аноним 28/01/21 Чтв 20:58:17 527654260
>>526419 (OP)
Надо вкатиться в магистратуру по молекулярной биологии с нуля, до этого было техническое направление.

Что посоветуете по литературе?
Думаю начать с тейлора, а вот по химии какую базовую литературу?
Аноним 28/01/21 Чтв 21:42:56 527659261
>>527632
В каком классе школы ты?
Правительствам не нужны нормальные люди в качестве граждан, им нужны тупые дегенераты и хуесос-минетчицы опущенные(рабочие).
Ими(правительствами) проводится отрицательная селекция с помощью налогов и законов.(материальная поддержка многодетных и прочих дегенератов, льготы, пособия, завоз мигрантов чурок и нигрилл, чек)
Аноним 28/01/21 Чтв 21:43:57 527660262
>>527627
Читай внимательнее, + или - похуй, главное что есть и то и другое и это разные штуки.
Аноним 28/01/21 Чтв 21:45:40 527662263
>>527660
Одинаковые. CPT симметрия тебе в помощь.
Шиз снова на связи Аноним 28/01/21 Чтв 22:18:47 527665264
Tachyon-200px.gif 220Кб, 200x56
200x56
ывфыввфывй.png 31Кб, 775x641
775x641
Увидел я значит гифку тахиона, где он при сверхсветовом движении распадается на части и это только подтвердило мои догадки. А догадка моя в том что не сверхсветовое движение невозможно, а невозможна классическая регистрация нашими приборами сверхсветового движения. В случае когда частота переваливает за гамм-предел - дальше у нас просто нет настолько точных антенн и мы видим картину фазового раздвоения и ухода в обратную сторону. Там где мы должны видеть две верхние точки волны - мы регистрируем одну. У нас просто нет материалов и методов регистрации квантов, движущихся со сверхсветовой скоростью и мы не додумались грамотно интерпретировать имеющуюся у нас билебирду, где мы получаем частоты в два и больше раз меньшую там где она не должна быть. Иными словами - предел скорости света это предел возможности измерить её.
Аноним 28/01/21 Чтв 22:23:29 527666265
Ах да я забыл напомнить что мои бредни не расходятся с теорией относительности. Теория относительности прекрасно и точно описывает волновую природу света (наших представлений о свете)
>>527665
Аноним 28/01/21 Чтв 22:59:09 527670266
>>527659
Не проецируй российскую действительность на остальные страны, "не школьник". Такое вполне можно реализовать, на мой взгляд, в развитых странах, а ещё проще в небольших, как, например, в северной Европе. Вопрос в том, как это сделать? Я не обладаю нужными знаниями, да и вообще не уверен в возможности это реализовать. Надеюсь здесь найдется какой-нибудь мимопсихолог или просто умный человек, который пояснит за это.
Аноним 28/01/21 Чтв 23:02:11 527671267
>>527665
Почему у тебя время дискретное.
Аноним 28/01/21 Чтв 23:10:19 527672268
>>527670
Ты просто наделаешь целое поколение людей, которые начнут убивать себя в возрасте 20-25 лет, из-за психологических проблем.
Аноним 28/01/21 Чтв 23:16:46 527674269
>>527672
А с чего это у них начнутся такие проблемы? Это наоборот заставит их понять себя, от чего и большинство псих.проблем они сами смогут решить или даже не возникнут.
Аноним 28/01/21 Чтв 23:19:46 527675270
>>527671
Это не время дискретное, а максимально возможный приём частоты антенной и аппаратурой.
Аноним 28/01/21 Чтв 23:25:28 527676271
>>527675
И какими физическими величинами у тебя ограничивается приём частоты. антенной блядь
Аноним 28/01/21 Чтв 23:28:33 527677272
>>527674
Самоконтроль это не про остановку сердца силой мысли. А значит и не про "решение своих псих проблем". Наши мозги работают на нейромедиаторах, и рецепторы имеют такую тенденцию, как привыкание. Излишний самоконтроль просто будет ломать тебе систему вознаграждения в больших количествах. Посмотри на детей из кадетских училищ, их там ебут самоконтролем с 7 до 21
Аноним 28/01/21 Чтв 23:48:35 527678273
>>527677
Там дисциплиной армейской ебут, я же говорю про наблюдение за своим психологическим состоянием, к примеру реакция на стресс, отслеживание эмоций и мыслей и т.д.
Аноним 28/01/21 Чтв 23:53:29 527679274
>>527678
Если я до этого просто думал что ты наивный, на такое я даже отвечать не хочу, это уже за гранью.
Аноним 28/01/21 Чтв 23:58:34 527680275
>>527676
>антенной блядь
Именно. Так то и рентгеновский снимок это тоже антенна. А красные черточки это.. предельно возможная нам известная материя, способная принимать как антенна гамма-излучение. Дальше частота просто идёт насквозь и никак себя не отображает, а если и отображает - мы по своей глупости воспринимаем её как излучение с меньшей частотой.
Аноним 29/01/21 Птн 04:49:46 527689276
Аноним 29/01/21 Птн 05:20:56 527690277
>>527670
>Такое вполне можно реализовать, на мой взгляд
Но только всем похуй на твой взгляд, тебя выебут в рот и в жопу и выкинут.
Аноним 29/01/21 Птн 05:21:38 527691278
>>527665
>Увидел я значит гифку тахиона, где он при сверхсветовом движении распадается
А кто снимал, бэтмен? или кусто?
Аноним 29/01/21 Птн 05:22:10 527692279
>>527662
>CPT
Счёт русско-татарский?
Аноним 29/01/21 Птн 09:06:22 527696280
>>527665
Хуйню какую-то высрал, дебил. Иди учи СТО.
Аноним 29/01/21 Птн 09:09:06 527697281
>>527690
Ты чего такой агрессивный?
Аноним 29/01/21 Птн 10:10:39 527700282
>>527501
Лол, похоже никто так и не смог посчитать.
Аноним 29/01/21 Птн 11:05:37 527704283
>>527700
>>527501
Да всем просто похуй. Иди смотреть лекции по квантовой электродинамике, на ютубе полно хороших лекций. Там всё есть, и рассказывают как такое считать.
Аноним 29/01/21 Птн 17:08:32 527730284
>>527192
>Есть модель для расчетов
Я разве это отрицал, потешный?

>>527231
>Магниты притягиваються потому что там так-сяк, кварки притягиваються потому что так-сяк. Планеты притягиваються потому что... вы не панимаете, эта другое!!!

И это я еще ебанутый, кек
Аноним 29/01/21 Птн 20:44:13 527736285
>>527632
>>527659
>>527670
>>527672
История воспитания вундеркиндов вряд ли могла состояться, если бы не отец девушек – педагог-психолог с еврейскими корнями Ласло Полгар. Именно он решил доказать миру, что задача воспитания гениев вполне доступна людям, правда, при достаточном желании.

Все началось в середине 1960-х годов, когда молодой Ласло Полгар, будучи тогда молодым педагогом-психологом, принял решение провести необычный эксперимент. Он задумал вырастить гениальное потомство. Тут следует учесть, что до оглашения своей задачи, он тщательно изучал биографии великих людей. На основе прочитанной информации психолог пришел к такому выводу: при длительной и кропотливой работе, а также высочайшей степени концентрации усилий, данная задача является вполне выполнимой.

В итоге выходило, что ребенок занимался каждый день не менее 8 часов, хотя занятия при необходимости могли затягиваться и на 11 часов. Удивительно, но работая в подобном режиме, Полгары за 7-10 дней успевали проходить годовые курсы начальной школы вплоть до 5 класса. Завистники заявляли, что родители заключили своих детей в домашнюю тюрьму, однако жизнь опровергла их заявления. Старшая дочка Полгаров в 12 лет возглавила женский шахматный рейтинг страны. Спустя 3 года она переместилась на первое

На достигнутом Жужа не остановилась, и в 1990 стала гроссмейстером в мужской версии шахмат. Вершиной достижений стало завоевание титула чемпионки мира в 1996. И хотя чиновники ФИДЕ по формальным причинам официально лишили ее данного звания, факт остается фактом – в 30 лет Жужа стала первой шахматисткой планеты. За старшей сестрой тянулись и младшие. В 1988 году они произвели настоящий фурор, когда все вместе приняли участие в шахматной олимпиаде среди женщин. Их командное семейное трио впервые

https://isralove.org/load/13-1-0-2856
Аноним 29/01/21 Птн 20:48:31 527737286
Электромагнитны[...].png 67Кб, 1280x685
1280x685
>>527696
А Доплера ты не забыл? Все что больше гаммы отрезается и воспринимается нами иначе. Сверхсвет это смещение спектра в половину и больше при потере плотности излучения в половину.
Аноним 29/01/21 Птн 20:52:12 527738287
>>527696
По моей гипотезе при движении источника света в сторону измерителя должна снижаться плотность излучения и появляться лишнее излучение в длинных спектрах, при этом скорость этого света равна скорости света и СТО не нарушается.
Аноним 29/01/21 Птн 20:52:22 527739288
>>527697
То тебе не ответ о моём отношении к тебе, а ответ об отношении мира а тебе.
Аноним 29/01/21 Птн 20:54:54 527740289
>>527736
Это немного не то, что я имел ввиду, но как пример неплохо.
Аноним 29/01/21 Птн 23:22:54 527744290
>>527738
Блядь.
В решениях уравнения поля есть неустранимая сингулярность для сверхсветовых движений носителей заряда. Она становиться более ясной при рассмотрение эм поля в спинорном представление. Короче, если ты хочешь увидишь эм взаимодействующий тахион, то придется сгенерировать разрыв в потенциале - а это соответствует эм волне с бесконечной энергий.
В квантовой случае еще хуже, нормировка летит полностью по пизде.
Аноним 30/01/21 Суб 04:03:22 527748291
>>527744
>В решениях уравнения поля есть неустранимая сингулярность для сверхсветовых движений носителей заряда. Она становиться более ясной при рассмотрение эм поля в спинорном представление.

Что за ахинею ты несешь?
Аноним 30/01/21 Суб 05:26:36 527749292
Двачик
Допустим в комнате 25 градусов тепла и допустим что стекло прозрачно для всего спектра излучения
На улице допустим -10
Комната будет излучать на улицу как почти абсолютно чёрное тело по спектру, соответствуя своей температуре 25 градусов
Но по мощности она сколько будет излучать через окно? Типа терять сколько энергии через окно излучением
Аноним 30/01/21 Суб 06:48:09 527751293
>>527730
Хуйло тупоголовое, иди общую теорию относительности учи. ОТО - теория гравитации. Знаешь, как ОТО описывает гравитацию? Нет? Тогда повторю еще раз: иди учи ОТО, сука ты ебанутая.
Аноним 30/01/21 Суб 06:52:56 527752294
>>527738
В жопу себе засунь свою "гипотезу". Я тебе говорю, учи СТО, из нее напрямую следует невозможность движения со сверхсветовой скоростью, хотя никакого прямого запрета специальная теория относительности на сверхсветовое движение не накладывает. Почему - поймешь, когда выучишь СТО. И с твоей высосанной из пальца "гипотезой" это не имеет ничего общего.
Аноним 30/01/21 Суб 07:35:15 527754295
Что можно почитать о советской псевдо/около-науке? ТРИЗ, мумиё, всякая магнитотерапия, историк Морозов, вот это всё. Прежде всего какие-то обыденные вещи в послевоенном СССР, не паранормальщина или ебля с макаками.
Хотелось бы не политических разоблачений, а нейтрального обзора и объянений, как это сосуществовало с реальной наукой.
Аноним 30/01/21 Суб 07:50:12 527755296
>>526419 (OP)
Есть смысл после 30 поступать на биофак, чтобы потом попробовать заниматься исследованиями? Это правда, что после 35 могут не взять в аспирантуру?
Аноним 30/01/21 Суб 10:06:53 527757297
>>527752
А я тебе отвечаю тем что Теория Относительности описывает исключительно волновую природу света, о корпускулярной там речи не идёт. Так что я не вижу противоречий. Максвелл тоже только волны описывал в контексте этой теории.
Аноним 30/01/21 Суб 10:18:20 527759298
>>527752
И о корпускулах и о формуле E=mc^2 я могу сказать ещё и то что кинетическая энергия существует только если в двух разных системах отчёта присутствуют тела. Если тело в одной системе отчета будет двигаться относительно пустоты в другой системе отчета то блять скорость этого тела может быть хоть бесконечной нахуй, но кинетическая энергия этого тела будет равна нулю.
Аноним 30/01/21 Суб 10:19:49 527760299
>>527752
Пока тело не столкнётся с другим телом или полем - можно считать что у тела вообще нет энергии.
Аноним 30/01/21 Суб 10:26:10 527761300
>>527757
Какую нахуй "волновую природу света"? Что ты, блять, несешь? СТО вообще не про это. СТО описывает пространство и время как единую структуру - пространство-время, описывает движение при скоростях, близких к скорости света. Природа света тут вообще ни при чем. Невозможность сверхсветового движения проистекает не из природы света, а из совершенно другой вещи. Ты вообще не шаришь. Ну и то, что мы называем "светом", - это лишь электромагнитное излучение в видимом для наших глаз диапазоне. Электромагнитное излучение - волновое, то есть распространяется волнами. Эти волны квантуются и получаются фотоны, но это не означает, что свет имеет "корпускулярную" природу. В современной физике элементарных частиц отошли от корпускулярно-волнового дуализма. Никто не называет частицы типа фотона или электрона волнами, а классические электромагнитные волны - потоком частиц. Частицы - это частицы, а волны - это волны, и, повторюсь, СТО тут совершенно ни при чем.
Аноним 30/01/21 Суб 10:45:15 527762301
>>527761
>Какую нахуй "волновую природу света"?
Ну которая берётся Эйнштейном у Максвелла и Лоренца в СТО. Это всё про волны блять. Дальше не читал.
Аноним 30/01/21 Суб 10:48:52 527763302
>>527761
>В современной физике элементарных частиц
В современной физике элементарных частиц статус фотона и электрона вообще под вопросом.
Аноним 30/01/21 Суб 10:53:53 527764303
>>527761
>В современной физике элементарных частиц
А ещё современная квантовая физика возродила эфир ололо когда пояснила что вакуум это вообще не вакуум и он заполнен под завязку частицами, которые постоянно возникают и исчезают.
Аноним 30/01/21 Суб 11:01:49 527765304
>>527761
Как бы тебе сказать так. Это база ёпт. Смотри, Специальная теория относительности построена на математике Максвелла и Лоренца, которые описывали волны. Тоесть теория сформировалась в контексте описания волновой природы электромагнетизма. Тоесть когда ты предлагаешь теорией, которая описывает волновую природу описывать не волны а что то другое - ты путаешь причину и следствие. Если свет больше не волна - тогда причем тут вообще теория относительно и её математика?
https://www.youtube.com/watch?v=ejNGCWEgJNU
Аноним 30/01/21 Суб 11:04:27 527766305
>>527762
Ты бредишь, ебанатик ты ебанутый. Иди проспись.
>>527763
И снова бредишь.
>>527764
Лол, пиздец ты поехавший. Ну или это такой жирный троллинг тупостью.
>>527765
Ебать поток сознания.
Аноним 30/01/21 Суб 11:08:17 527767306
>>527766
Ты бредишь сука ебаная. Ты путаешь причину и следствие в голове у себя.
Аноним 30/01/21 Суб 11:10:10 527768307
>>527766
Я тебе ещё раз поясняю что теория относительности была сформирована во время корпускулярно-волнового дуализма. Если ты его вынимаешь из теории относительности - ты блять кирпичи из фундамента здания вытаскиваешь.
Аноним 30/01/21 Суб 11:13:05 527769308
>>527766
Давай предложи ещё мне использовать формулу E=mc^2 для описания безмассовых частиц ёпт.
Аноним 30/01/21 Суб 11:17:13 527770309
>>527766
>Ебать поток сознания
Ебал я в рот вспоминать Лоренцовую алгебру и эти ебучие координатные сетки из СТО для всей этой хуйни (хотя там довольно простая математика). Хотя временной парадокс в диаграмме Лоеделя для трех движущихся тел с околосветовой скоростью меня приколол.
Аноним 30/01/21 Суб 11:23:55 527771310
>>527766
Понимаешь я много чего то не помнить и что то не понимать. Но я не пришёл на наукач чтобы всех обсирать и называть шизами. Я пришел мирно подискутировать. Я не отношусь к отрицателям Теории Относительности, которые считают что Эйнштейн был жидомасоном-русофобом, который хотел всех наебать.
Аноним 30/01/21 Суб 13:49:03 527772311
>>527767
Ясно.
>>527768
Еблан, СТО вообще не про природу света. Сколько раз тебе нужно это повторить, чтобы до тебя дошло?
>>527769
При чем тут эта формула, дегенерат? Она описывает эквивалентность массы и энергии. Энергию можно перевести в массу и наоборот. При чем тут какие-то частицы? Что ты несешь, поехавший?
>>527771
Блять, ты реально ебанутый на всю голову.
Аноним 30/01/21 Суб 14:45:54 527773312
Возможно ли доставать питательные вещества прямо? Взял ты, например, пару телег грязи, запихнул вспомогательные соли, пропускаешь через какой-нибудь аппарат, и получаешь на выходе питательный сок.
Аноним 30/01/21 Суб 15:10:30 527774313
>>527773
Возможно. К примеру сахар. Добыча сахара человеком - добыча питательных веществ из того в чем они есть. Производство сахаров растениями - добыча питательных веществ из всякого гавна.
Аноним 30/01/21 Суб 23:23:17 527784314
Есть один момент, который я так и не понял с курса физики. Первый закон ньютона говорит, что сила равняется массе на ускорение. Но если я давлю пальцем на стену, то сила есть, а определить её по этой формуле нельзя, поскольку палец никуда не двигается и ускорения нет. Что я упускаю?
Аноним 30/01/21 Суб 23:27:37 527785315
>>527784
Упускаешь силу, которая действует на твои ноги, сила трения, которая возникает, когда ты давишь пальцем на стену.
Аноним 30/01/21 Суб 23:29:11 527786316
>>527785
Я вообще не о том, а о первом законе ньютона. Сила есть, хотя ускорения на пальце нет, формула не работает.
Аноним 30/01/21 Суб 23:31:13 527787317
>>527786
Я тебе говорю еще раз, надави пальцем на стену стоя в комнате, с абсолютно скользким полом.
Аноним 30/01/21 Суб 23:33:47 527789318
>>527787
А я тебе ещё раз повторяю, я стою не на скользком полу и никуда не двигаюсь относительно стены, ускорение равно нулю. Чему равна сила?
Аноним 30/01/21 Суб 23:34:36 527790319
>>527789
Суммарная сила, действующая на тебя равно нулю.
Аноним 30/01/21 Суб 23:36:48 527791320
>>527790
Я не про силу на меня спрашивал, зачем ты виляешь? Посчитай силу, с которой я давлю пальцем на стену.
Аноним 30/01/21 Суб 23:37:50 527792321
>>527791
Посчитал, ноль. пожимает плечами и недоуменно смотрит
Аноним 30/01/21 Суб 23:41:21 527793322
>>527792
Это неправда, зачем ты мне врешь? Сила, с которой я давлю пальцем на стену не равняется нулю. Более того, материал стены испытывает напряжения связанные с силой и если материал стены это плёнка, то стена может продавиться. Из-за силы, которая по твоему равняется нулю. Так что подожду того, кто знает ответ.

>>527784
Бамп вопросу.
Аноним 30/01/21 Суб 23:43:10 527794323
>>527793
На твой палец действуют две силы. Первая, сила реакции опоры. Вторая, сила, с которой кисть давит на палец. Обе одинаковые и направлены друг против друга.
Ситуация не отличается от того, если бы ты давил на стену карандашом. На него ровно две силы бы действовало.
Аноним 30/01/21 Суб 23:46:15 527795324
>>527794
Повторяю, я не спрашивал про силу на свой палец. Хватит мне писать, очевидно ты знаешь ещё меньше меня. Больше тебе не отвечу, буду бампать свой вопрос пока не получу ответа на СВОЙ вопрос.
Аноним 30/01/21 Суб 23:48:43 527796325
153e5a7affb00c5[...].jpg 42Кб, 640x360
640x360
Аноним 30/01/21 Суб 23:48:52 527797326
Аноним 31/01/21 Вск 00:36:32 527800327
Нахуя вообще нужна математика? Я конечно проёбывал 90% занятий в школе, но по физике я не помню, чтобы мне пригодились уравнения. Просто унылый дроч по зазубренной формуле, подставляя данные из условия значения под эти самые формулы. Где применяется математика? Объясните на пальцах, плиз, где я могу применить какую хуйню вроде уравнений, модулей, графиков? Для чего это?
Аноним 31/01/21 Вск 00:39:55 527801328
>>527800
Математика это инструмент, как молоток или плоскогубцы. И этот инструмент используется по назначению в естественных науках. Если тебе по жизни не нужен молоток это не значит, что этот инструмент бесполезный.
Аноним 31/01/21 Вск 00:53:28 527802329
>>527800
Если тебе понадобится призвать демона, то надо будет чертить гексаграмму, а без математики никак. С алхимией и астрологией то же самое.
Аноним 31/01/21 Вск 00:55:32 527803330
>>526419 (OP)
Я хз где спросить, я просто в ебаном отчаянии уже, но по сути это подпадает под категорию тупых вопросов.
Кароче, вопрос по математике.
Я хочу реализовать триангуляцию в одной онлайн игрушке, суть в том что на 3 моих аккаунта приходит информация о расстоянии между некой точкой и самим этим аккаунтом в игровом мире.
Координаты моих аккаунтов в мире известны, и они расставлены в виде треугольника в концах этой игровой карты.

Сначала я воспользовался методом поиска примерной зоны путём нахождения точек пересечения трёх окружностей, тоесть сначала находил вообще все точки пересечения окружностей с друг другом, а потом искал три точки, которые были в радиусе других окружностей, и это работало для двухмерной плоскости.

Вся проблема в трёхмерном пространстве, я думал использовать вместо окружностей сферы - но как мне найти точки пересечения сфер? Вот допустим я нашёл центральную точку пересечения сфер и радиус (точкой пересечения сфер является окружность), как мне теперь построить окружность в трёхмерном пространстве?

Написал по ублюдски но я реально уже в отчаянии, надеюсь что кто нибудь мне поможет:(
Аноним 31/01/21 Вск 01:23:34 527806331
не нашел треда вкатывающихся в физику
аноны, решал сегодня школьную задачку: поезд движется по окружности радиуса 100м, с какой скоростью он должен двигаться, чтобы шарик, расположенный внутри поезда, составил угол 45 градусов с вертикалью
решение:
поезд движется равномерно => тангенциальное ускорение отсутствует. Но сила трения имеется, F_тр = mv2 / R
Тогда на поезд при движении действуют 2 силы: mv
2 / R - направленная к центру окружности И сила тяжести mg. Тогда скорость находится из mv2 / R = mg
А вот тут вопрос. Эти силы действуют на поезд - ок. Но как понять, почему mg действует И на шарик ВНУТРИ поезда?
На практике меня смущает следующая аналогия: предположим я встану на огромную гору, на меня же НЕ действует сила тяжести Mg, где M - масса горы?
Можно ведь сделать поезд без крыши, провести тонкую ленту у вершины поезда, а на нее подвесить шарик. Массу поезда будем очень сильно наращивать и, исходя из решения, сила тяжести, действующая на шарик, будет РАСТИ
Аноним 31/01/21 Вск 01:28:05 527808332
>>527803
>как мне теперь построить окружность в трёхмерном пространстве?
Зачем так сложно, если можно просто сразу захуярить в программу аналитическое решение с бумаги?
У тебя всё сведется к банальной системе линейных уравнений
Ax=b, матрица A и вектор b находятся довольно просто, я сейчас за несколько минут привел систему к такому виду.
Аноним 31/01/21 Вск 01:42:44 527810333
>>527808
Я в математике не разбираюсь совсем, не мог бы пожалуйста подробнее написать что мне надо делать?
Аноним 31/01/21 Вск 01:47:09 527811334
image.png 42Кб, 498x276
498x276
>>527810
Ну давай начнем с конца. Если тебе линейная система уравнений дана, как решать будешь?
Линейная, как на пике
Аноним 31/01/21 Вск 01:54:54 527813335
>>527811
>Если тебе линейная система уравнений дана, как решать будешь?
Мои познания математики остались где то на уровне 5 мб 6 класса школьной программы, я хз чё это ваще такое на пике, выглядит страшно.
Аноним 31/01/21 Вск 01:57:38 527814336
>>527813
Ну тогда тут gg wp. Можно было бы объяснить что тебе сделать, но займет часы.

P.S. Ребята, хотите игры программировать, учите линейную алгебру.
Аноним 31/01/21 Вск 02:01:38 527816337
>>527814
Распиши хоть как нибудь пожалуйста, а про системы уравнений и т.д я нагуглю потом и вникну, мне щас хотя бы просто понять куда копать.
Аноним 31/01/21 Вск 02:36:51 527817338
>>527816
Если тебе не хочешь связываться с системами уравнений и не нужно аналитически решать, то можно просто подбором. В ексель или гуглдоки загоняешь три уравнения вида (x-x0)^2+(y-y0)^2+(z-z0)^2+R^2 где (x0;y0;z0) - центр твоего аккаунта, R - соответствующее растояние. Потом подбираешь x,y,z такие чтобы все три уравнения были нулями. В ексель и гугл доках (тут не уверен в функционале) есть реализации методов оптимизации которые могут подобрать за тебя (так как у тебя получилась задача нахождения минимума). Только учти что в результате будет две точки (но найдешь ты только одну - нужно будет менять стартовую точку пока не найдешь вторую). Для поиска точки в пространстве нужно 4 сферы. Кстати для системы уравнений тоже нужно 4 уравнения (тоесть 4 аккаунт).
Аноним 31/01/21 Вск 03:30:31 527820339
image.png 6Кб, 360x102
360x102
>>527817
Ну я пытался сперва всё решить системами уравнений, увидел на одном форуме решение проблемы которая мне нужна (пикрил), нашёл в интернете какой то калькулятор и просто вбил туда значения, но суть в том что расстояние передаётся округлённое, то есть оно не точное, и калькулятор для округленных значений расстояния мне выдавал что нету решений.

А для уравнений которые ты скинул как я понял тоже нужны точные значения.
Аноним 31/01/21 Вск 03:46:24 527821340
Сколько в часах займет прочтение всех доступных литературных произведений и учебников? Если скорость чтения — "качественное усвоение", не дебильное скорочтение.
Аноним 31/01/21 Вск 04:16:50 527823341
>>527784
Прикинь, это не единственная формула силы
Вот тебе более подходящая:сила давящего гидроцилиндра равна площадь поршня умножить на давление
Аноним 31/01/21 Вск 04:25:11 527824342
>>527823
Давление считается как сила на площадь. Главная здесь сила, и закон есть как раз для неё. Но это все хуйня, ты продолжаешь маневрировать и не отвечать на вопрос.

Сила у меня приложена точечно к стене. Дай мне формулу, чтобы найти эту силу.
Аноним 31/01/21 Вск 06:07:27 527828343
>>527824
Ты очень тупой, продолжай думать о цилиндре пока не дойдет.
Аноним 31/01/21 Вск 10:57:08 527834344
Есть отдельный жанр медиа-контента "люди тупые". Опрашиваемые журналистами прхожие не знают Гагарина и Наполеона, участники теле-квизов проваливаются на элементарных вопросах, регулярно выходят статьи типа "95% людей не знают почему сменяются времена года".
Так вот, насколько население РФ или США действительно невежественно, как это оценить?
Аноним 31/01/21 Вск 11:34:53 527835345
>>527834
>Так вот, насколько население РФ или США действительно невежественно, как это оценить?
Ты наверное не про невежество, а про тупость все-таки? Тут несколько проблем

>95% людей не знают почему сменяются времена года.
А не всем даже это объясняли по школьной программе. Мне например не объясняли.

Прикинь какой-нибудь вопрос вроде этого:

Расположите планеты в порядке последовательности по размеру, от меньшей к большей (планеты: Юпитер, Сатурн, Меркурий, Нептун).

Многие ли ответят в России? Сомневаюсь, астрономии в школах не было толком, да и сейчас почти нет.

Что до тестирования. Ну можно всероссийскую аттестацию сделать единоразовую. Прогон по усредненным анкетам на 200-400 вопросов из стандартной программы образовательной, на каждом рабочем месте условно, в обязательном порядке.

Поотбирать телефоны на момент прохождения тестирования, сами бланки с вопросами пломбировать и всё такое. Все-равно сольют конечно, но более или менее конкретный результат получим. Ну и проверка всех бланков централизованная нужна (в одном месте).

Аноним 31/01/21 Вск 11:40:40 527837346
Помню про эксперимент с мышью. Мыши вставляли электрод в голову, чтобы стимулировать центр удовольствия. Она должна была нажимать на кнопку, чтобы получить кайф. Говорят, что в результате она постоянно жала кнопку и умерла от голода. Хз правда ли это, но вопрос в другом.
Был ли эксперимент где мыши или другому животному вставляли такой электрод и на потоянке стимулировали центр удоволсьствия(то бишь мышь пермаментно кайфовала) без необходимости нажатия кнопки или совершения любых других действий? То есть мышь могла бы делать что хочет и кайф бы был все равно. Если такой эксперимент был, то чем он закончился: что делала мышь, или она потеряла полностью мотивацию к действиям?
Аноним 31/01/21 Вск 12:07:16 527838347
>>527837
А сам догадаться не можешь?
Аноним 31/01/21 Вск 12:22:07 527840348
>>527838
Я могу догадаться что ты результат моего секса с твоей мамашей 7 лет назад. Эксперимент чтобы проверить это проводить мне не нужно.
Аноним 31/01/21 Вск 12:42:13 527841349
>>527821
Невозможно определить, т.к. абсолютно не представляется возможным подсчитать и учесть все имеющиеся произведения и учебники. Кроме того, скорость чтения - субьективна, что делает вопрос ещё более бессмысленным.
Аноним 31/01/21 Вск 12:43:55 527842350
Аноним 31/01/21 Вск 13:00:22 527844351
combined.png 9Кб, 640x480
640x480
В бензине для машин до сих пор используется свинец? Пик, 5-6, это снег с крыши многоэтажки, не выпаривал; 11-12 графики, это вода из крана. Чем больше пик, тем выше концентрация, но не знаю сколько в граммах/литр, так как нет эталона. по два графика, это разная продолжительность анализа, чем дольше, тем меньшую концентрацию можно выявить. Снег пролежал всего дня 3-5. Другие источники, это сжигание угля, но у нас ТЭЦ сжигает газ, вроде бы, но есть частные дома.
Попробую за городом и возле трассы собрать. Интересно если кто-то регулярно проводит такие анализы по регионам.
Это из Молдавии, Кишинев.
Аноним 31/01/21 Вск 13:28:47 527846352
>>527820
> расстояние передаётся округлённое
Тогда нужна оптимизация. Только ищешь не ноль, а минимизируешь сумму квадратов уравнений (x-x0)^2+(y-y0)^2+(z-z0)^2-R^2 (в моем прошлом посту я проебал один минус). Гкглдоки не справляются с таким (им нужны линейные). Нужен или ексель или какой-то пакет способный на не линейные оптимизации.
Аноним 31/01/21 Вск 13:35:19 527847353
>>527784
Бамп вопросу, никто за день так и не ответил по существу.
Аноним 31/01/21 Вск 14:51:27 527851354
Аноним 31/01/21 Вск 15:31:26 527852355
>>527847
Начни с того что прочитай первый закон ньютона. Никто не будет комментировать выдуманые тобой законы.
Аноним 31/01/21 Вск 15:40:04 527853356
>>527801
Так я это понимаю, но всё же мне хочется иметь представление о том, где именно я эти хуевы уравнения.
Не захуй оно мне нужно в целом, а просто где я могу их применить, для понимания пользы математики. Твой ответ этож просто считай слова вроде "она полезна, но не для тебя, пошёл нахуй", а мне надо понять, где я могу какую-то конкретную хуету вроде тех же, блять, уравнений применить. Не в жизни, а вот просто вне обучения математики, практичная польза, но необязательно что именно для меня, быдла, это будет полезно.

>>527802
Вот это примерно тот ответ, что я ждал, спасибо. Но всё же хотелось бы узнать не пространное "без математики никак", а что-то вроде "без системы уравнений в данном случае никак".
Аноним 31/01/21 Вск 15:41:09 527854357
>>527852
Уже читал, F = ma это следствие первого закона. В моем примере a равно нулю, а сила не равна нулю. Теперь пиздуй перечитывать мой первый пост, я именно об этом моменте и спрашивал.

>>527784
Бамп вопросу, так никто и не смог ответить.
Аноним 31/01/21 Вск 15:42:48 527855358
>>527854
Ускорение твоего пальца?
Аноним 31/01/21 Вск 15:44:27 527856359
>>527855
Ускорение моего пальца относительно стены, да.
Аноним 31/01/21 Вск 15:48:31 527857360
>>527856
У тебя стена плохо двигается, значит
Аноним 31/01/21 Вск 15:51:51 527858361
>>527857
Потому и нет ускорения
Аноним 31/01/21 Вск 15:52:22 527859362
Аноним 31/01/21 Вск 15:54:29 527861363
>>527859
Всё очевидно. Ты пытаешься передвинуть палец, но поскольку его масса равна нулю, то и сила равна тоже нулю.
Аноним 31/01/21 Вск 15:54:53 527862364
>>527854
> читал, F = ma это следствие первого закона
Выкинь свои фрические книжки и читай нормальные - где описаны настоящие законы ньютона. Если ты (или какой-то фрик) выдумал хуиту которую назвал "первым законом" то хуита у тебя дальше получаться и будет.
Аноним 31/01/21 Вск 15:55:35 527863365
>>527861
Сила не равна нулю, стена в месте соприкосновения испытывает напряжения из-за приложенной внешней силы.
Аноним 31/01/21 Вск 15:56:13 527864366
Аноним 31/01/21 Вск 15:57:02 527865367
>>527863
А стена двигается?
Аноним 31/01/21 Вск 15:57:38 527866368
>>527865
Нахуй иди, чмошник. Я здесь задаю вопросы.

>>527784
Бамп вопросу.
Аноним 31/01/21 Вск 15:59:14 527867369
>>527866
Начни лучше с определения понятия силы. Я же просто пытаюсь прояснить ситуацию
Аноним 31/01/21 Вск 16:00:49 527868370
>>527867
Я пришёл в тред тупых вопросов за своим ответом, если ты не в состоянии его дать, то проходи мимо. Если бы мне выше описанный момент был ясен, я бы не задавал вопрос.
Аноним 31/01/21 Вск 16:01:39 527869371
>>527868
> пришёл в тред тупых вопросов за своим ответом
Никто тебе про выдуманые тобой законы ответить не сможет.
Аноним 31/01/21 Вск 16:01:56 527870372
>>527866
Кстати, если будешь всех посылать лесом, то тебе никто не будет отвечать, подумай хорошенько
Аноним 31/01/21 Вск 16:02:44 527871373
>>527870
Я никого не посылал до теперь, но мне никто так и не ответил. Не заметил разницы.
Аноним 31/01/21 Вск 16:02:56 527872374
>>527868
Начни с определения понятия силы
Аноним 31/01/21 Вск 16:04:37 527873375
>>527872
> Общепринятое определение силы отсутствует
Аноним 31/01/21 Вск 16:05:37 527874376
>>527873
Источник?
Если это так, то твой вопрос не имеет того смысла, на который ты возлагаешь
Аноним 31/01/21 Вск 16:06:03 527875377
>>527871
Тебе ответили с предложением почитать настощие законы ньютона, с последующим объяснением непонятных тебе моментов. Я подозреваю что законы ты таки прочитал, то что ты хуйню пишешь понял, но вместо того чтобы просто уйти начал зачем-то срать в тред.
Аноним 31/01/21 Вск 16:09:18 527876378
>>527875
Прочитал, но не понял.

>>527874
Википедия, а какие источники у неё я не смотрел.
Аноним 31/01/21 Вск 16:11:25 527877379
>>527876
К чему прикладывается сила?
Аноним 31/01/21 Вск 16:15:35 527878380
>>527876
> Прочитал, но не понял.
Тебе нужен не первый а второй закон ньютона. Буква F в F = ma это не сила а сумма всех сил действующих на материальную точку. Если ты давишь палцем не стену а она не двигается это значит что сумма сил равна нулю, и на стену действует еще какая-то сила равная силе твоего пальца (и противоположная по действию). К примеру сила реакции опоры (но у стены там дахуя чего может быть).
Аноним 31/01/21 Вск 16:26:27 527879381
Масса и ускорение тела во втором законе Ньютона относятся не к телу, что прикладываю (палец), а к прикладываемому (стена).
Если я возьму шарик (прикладываемое) и начну его двигать пальцем, то F=ma будет описывать поведение шарика.
Аноним 31/01/21 Вск 16:29:13 527880382
>>527879
Смелое заявление. Учитывая третий закон.
Аноним 31/01/21 Вск 16:33:59 527881383
>>527878
Из этого следует, что сила приложенная пальцем равняется силе реакции опоры по значению и противоположна по знаку. Эти силы не равны нулю. Тогда как найти силу приложенную пальцем?
Аноним 31/01/21 Вск 16:35:20 527882384
>>527880
Противоречия?
Было сказано лишь то, что F=ma относится к одной материальной точке
Аноним 31/01/21 Вск 16:43:08 527883385
квадратное уура[...].mp4 39917Кб, 1920x1080, 00:03:48
1920x1080
Аноним 31/01/21 Вск 16:44:17 527884386
маца Ньютона.mp4 24457Кб, 1280x720, 00:02:42
1280x720
Аноним 31/01/21 Вск 16:46:13 527885387
>>527881
По косвеным признакам. Например степени деформации стены либо твоего пальца.
>>527882
К обоим (естественно для каждого своя). Просто в случае шарика на него кроме пальца действуют очень слабые силы их сумма очень мало отличается от силы пальца и следовательно его можно двигать пальцем. Но точно с такой же силой шарик давит и на палец. Но сил воздействующих на палец достаточно чтобы обнулить эту силу. Если бы на тебя не действовало никаких других сил кроме шарика (невесомость без какой-либо опоры) попытка его подвинуть вызвала бы интересные переживания. Естественно разница в массе тел дает огромное преимущество владельцу пальца, но современем и он почувствует ускорение от шарика.
Аноним 31/01/21 Вск 16:47:01 527886388
>>527879
>Масса и ускорение тела во втором законе Ньютона относятся не к телу, что прикладываю (палец), а к прикладываемому (стена).

Ошибаюсь.

Но это сути не меняет. F=ma относится лишь к одной материальной точке.
Аноним 31/01/21 Вск 16:49:16 527887389
>>527885
> По косвеным признакам. Например степени деформации стены либо твоего пальца.
Это точно единственный способ? Ты не решил меня наебать как те, кто заявлял, тсо сила равна нулю?
Аноним 31/01/21 Вск 16:54:47 527888390
>>527887
> Это точно единственный способ?
Ну еще если на тело действует только одна сила (либо остальными силами можно пренебречь) то так ты хотел сразу - считать массу и ускорение.
Аноним 31/01/21 Вск 17:10:17 527889391
>>527784
>Первый закон Ньютона говорит, что сила равняется массе на ускорение

Это гласит лишь само определение силы.

Первый закон Ньютона - это о том, что если на тело не приложена никакая сила, то оно либо покоится, либо двигается с постоянной скоростью (равномерно и прямолинейно).

Второй закон Ньютона - сумма всех сил, приложенных к телу, есть тоже некая сила (результирующая) и равна произведению массы на ускорение этого же самого тела под действием других сил.

Третий закон Ньютона - силы взаимодействий двух тел противоположны друг другу, но равны по модулю.


>Но если я давлю пальцем на стену, то сила есть...

Ибо второй закон Ньютона

>Но палец никуда не двигается

И не должен ты его просто сдерживаешь своими мышцами. Ты должен рассматривать не палец, а стену, потому что ты прикладываешь силу (посредством пальца) к стене, а не к пальцу.

Аноним 31/01/21 Вск 21:07:33 527903392
Аноним 01/02/21 Пнд 00:36:52 527923393
>>527834
>действительно невежественно, как это оценить?
По объёму экспорта НЕ ПРИРОДНЫХ РЕСУРСОВ в отношении на человека.
Аноним 01/02/21 Пнд 00:38:15 527924394
Аноним 01/02/21 Пнд 14:05:58 527945395
Аноним 01/02/21 Пнд 18:18:31 527951396
>>527945
Мышь умрёт от передоза собственных медиаторов, мамоёб малолетний. Задолго до того, как умрёт мышь с кнопкой. Анус себе побампай, это твой единственный способ получить секс.
Аноним 01/02/21 Пнд 18:42:39 527954397
Какие есть способы получить единичный фотон в "комнатных" условиях? Не обязательно сам источник его должен излучать. Например, могу ли я взять обычный лазер и черное стекло/фильтр типо тонировки, зная мощность, длину волны, "черность", получить после при пропускании через это стекло/фильтр один единственный фотон?
Аноним 01/02/21 Пнд 19:32:32 527958398
>>527954
Да, можно. Но нужно будет аккуратно подбирать, и там будет конечно же колебания, порой два порой три.
Аноним 01/02/21 Пнд 21:22:24 527962399
>>527954
На коленке не получиться.
По идеи можно сделать на полупроводником лазере, но его надо сильно охладить, да еще к специальному источнику питания подключать.
Аноним 01/02/21 Пнд 21:40:11 527965400
>>527954
Я тут недавно этим интересовался, налетели квантовошизикопетухи написало пару возможно разбирающихся человек, и сказали что возможно они говорили точно, но что они понимают даже "один" фотон это не один фотон, а "пакет", пока не попал в цель, неопределённого состава, неопределённой длины волны, или их там много, а может это и одно и то же и т.д.
Аноним 02/02/21 Втр 07:42:09 527985401
>>527965
Может под неопределённым составом имелось в виду состав какого-нибудь источника света, лампочка например? Хотя да, и в лазерах разброс по длине волны присутствует.
>>527962
Поэтому и написал пример со стеклом, подразумевая хотя бы возможность фильтрации, ибо получить пучок из десятка фотонов уже не триллиард в поперечнике, как у того же маломощного красного лазера. Интересует именно присутствие какого-то фундаментального ограничения на такие вот приколы
Аноним 02/02/21 Втр 08:23:18 527987402
unnamed.gif 1Кб, 459x179
459x179
>>527954
Нет, сейчас ещё не умеют делать однофотонный источники. Самое близкое - излучение одиночной квантовой точки, но опять же, оно изучит не один фотон, а суперпозиция нуля и одного фотона + шум от канала.

>>527958
Так вообще получится когерентное состояние, где амплитуды распределены по пуассону(см пикрил)

>>527985
Фундаментальные ограничения - в случае с лазером и стеклышком излучение описывается уравнением в пикриле. Если на качественном уровне - у тебя нельзя отправить один фотон и быть уверенным, что ты отправил именно 1, а не 0 или 2.
Аноним 02/02/21 Втр 09:00:05 527988403
>>527987
Спасибо за ответ, но можешь объяснить значения параметров в уравнении, если мы рассматриваем лазер и стёклышко?
Аноним 02/02/21 Втр 11:58:33 527994404
Как работает заземление? Ток действительно проходит от одного штыря вбитого в землю по сырой почве к другому штырю?
Аноним 02/02/21 Втр 12:06:50 527997405
Аноним 02/02/21 Втр 12:46:27 528001406
Аноним 02/02/21 Втр 14:10:58 528007407
16120532658290s.jpg 5Кб, 200x173
200x173
Почему до сих пор не сделали такую машину?
Аноним 02/02/21 Втр 14:18:24 528008408
>>527987
>Так вообще получится когерентное состояние, где амплитуды распределены по пуассону(см пикрил)
Ты это сам сейчас придумал? Да и это не особо противоречит идеи об одиночном фотоне, никто не же говорил, что фотон должен обладать конкретной энергией
Аноним 02/02/21 Втр 14:27:50 528009409
>>528007
А зачем?
Энергия притяжения компенсируется энергией деформации балки не которой закреплен магнит.
Аноним 02/02/21 Втр 14:34:36 528010410
Аноним 02/02/21 Втр 14:37:38 528011411
>>527988
Альфа - среднее кол-во фотонов.
Лазер создаёт состояние с большой альфой, стеклышко уменьшает альфу в "черноту" раз

>>528008
>не противоречит идее об одиночном фотоне
Оно противоречит идее об получении одиночного фотона лазером и стеклом. О невозможности реализовать это хоть как-то оно ничего не говорит, только про лазер и стеклышко.
Аноним 02/02/21 Втр 14:50:07 528014412
>>528007
имхо вся работа проводится в изолированной от мировой системе отчёта, то бишь работа на колёса, то что связывает данную систему отчёта с мировой не будет поступать, но возможно я не прав
Аноним 02/02/21 Втр 15:45:07 528019413
Что такое "спиновой ток"?
Аноним 02/02/21 Втр 15:46:11 528020414
>>528011
>Оно противоречит идее об получении одиночного фотона лазером и стеклом. О невозможности реализовать это хоть как-то оно ничего не говорит, только про лазер и стеклышко.
А, всё я понял о чём ты. Я сейчас посмотрел еще раз на то что ты прикрепил, да, мои слова про "там будет то 0 то 1 то 2 то 3" действительно про Пуассоновское распределение, выводится же из квантовой оптики в две строчки, через кинетику. Я просто тупанул и почему-то увидел там операторы рождения вместо альфы и немного прихуел от формулы.
Я правильно понимаю? Что остается только делать эксперимент с этой хуйней и грамотно отбирать только те случаи, где прилетел только один фотон. Если же нужно, чтобы в эксперимент залетало состояние ровно |1>, то придется видимо ебаться с получением запутанных фотонов, и в одной части установки измерять их количество (создавая смешанное состояние из чистого), а в другой части установки делать уже свой эксперимент, после чего отбирать только результаты тех попыток, которые в первой части установки давали 1 фотон?
Аноним 02/02/21 Втр 15:46:46 528021415
>>528019
Электрический ток, движение заряда. Спиновый ток, движение спина.
Аноним 02/02/21 Втр 15:48:59 528023416
>>528021
Спиновой ток = ток электронов? Но почему его тогда называют спиновой?
Аноним 02/02/21 Втр 15:54:04 528024417
>>528019
Тоже самое что векторный ток, только вместо векторов спиноры.
Короче у нас гладкое многообразие меняется на произведение гладких многообразий.
Аноним 02/02/21 Втр 15:57:55 528025418
>>528024
>спиноры
Спиннеры?
Аноним 02/02/21 Втр 15:58:16 528026419
>>528023
Спиновый ток это про движение частиц со спином, либо может быть вообще без движения электронов. Если у тебя в цепочке электронов спины решили сделать волну, то будет спиновый ток. Ну это не только электроны могут быть. Ну и эта штука явно отличается от обычного тока. Заряд вещь скалярная (число короче), поэтому у тока есть аналогия с течением жидкости. Спин же, это что-то типа вектора (не совсем конечно, там спинор). Я попробую пояснить тебе на аналогии в чём ты ошибаешься, ведь ты даже с обычным электрическим током ошибаешься.

Вот смотри, представь себе проходную где люди идут. У тебя там условно идет 100 человек в минуту. У каждого человека в кармане есть деньги, а значит через проходную есть и поток денег. Вот ты в случае с электрическим током и людьми, путаешь поток людей и поток денег. Оно и понятно почему, ведь у всех электронов одинаковый заряд, это как если бы у всех в кармане лежало одинаковое количество денег. Но всё же, это разные понятия.
Теперь по поводу спинового тока, представь что у тебя у всех этих людей есть часы, у кого-то спешат у кого-то бегут. Но если представишь стрелку часов как вектор и будешь его складывать от всех людей, то увидишь, что у тебя есть поток этого вектора. Это вот что-то похожее на спиновый ток.
Аноним 02/02/21 Втр 16:00:01 528027420
>>528026
А что такое спин?
Аноним 02/02/21 Втр 16:00:20 528028421
Аноним 02/02/21 Втр 16:00:33 528029422
Аноним 02/02/21 Втр 16:02:05 528030423
>>528029
А если своими словами?
Аноним 02/02/21 Втр 16:04:53 528032424
>>528027
Состояние квантовых систем образуют сложное множество, которое вроде линейное, но в нем есть однозначность. Спин это затычка этой неоднозначности.
Аноним 02/02/21 Втр 16:04:54 528033425
>>528027
Величина у частиц, которую открыли где-то лет 100 назад. Знали что есть масса и заряд, но наткнулись на спин. В отличие от массы и заряда, спин штука странная и имеет направления. Связано с симметрией частицы по отношению к вращению системы координат. Его в классических аналогиях сложно объяснять.
Аноним 02/02/21 Втр 16:07:16 528034426
>>528032
А по человечески объяснить можно? Это что такое?
Аноним 02/02/21 Втр 16:08:02 528035427
>>528033
>симметрией частицы
А как узнали что частица типа несимметричная?
Аноним 02/02/21 Втр 16:11:10 528036428
>>528030
В какую сторону торчит электрон.

Но это уебанское объяснение. Привыкай, что в квантах/ядерной физике квантовые числа вводят часто и обывательского смысла они не несут, это просто ещё одна степень свободы.
Аноним 02/02/21 Втр 16:11:08 528037429
>>528033
>Его в классических аналогиях сложно объяснять.
Но вот как ты сам лично для себя его представляешь?
Аноним 02/02/21 Втр 16:12:05 528038430
>>528037
Как вектор в пространстве комплексных матриц.
Аноним 02/02/21 Втр 16:13:43 528039431
>>528034
Ну бля. Скорее нет. Тут нужно объяснять целый солидный кусок теории групп.
Аноним 02/02/21 Втр 16:15:12 528040432
>>528038
Но вектор это математическое, нереальное, а как ты представляешь спин у реального электрона?
Аноним 02/02/21 Втр 16:18:01 528041433
>>528040
Число пи тоже нереальное?
Аноним 02/02/21 Втр 16:21:44 528043434
>>528041
Ну да, это же число. Ну вот смотри, мы понимаем что такое масса частицы это типа как легко можно разогнать частицу или как трудно ее затормозить. Что такое заряд типа тоже понятно это как частицы притягиваются, как они перемещаясь переносят ток. А что такое спин?
Аноним 02/02/21 Втр 16:21:51 528044435
>>528040
А как ты себе представляешь аромат у кварка?
Аноним 02/02/21 Втр 16:23:00 528045436
>>528044
>А как ты себе представляешь аромат у кварка?
А ты?
Аноним 02/02/21 Втр 16:25:21 528046437
>>528045
Как ещё одну степень свободы системы.
Аноним 02/02/21 Втр 16:26:16 528047438
>>528046
А как ты представляешь себе "степени свободы"?
Аноним 02/02/21 Втр 16:27:00 528048439
>>528047
Как ещё одну размерность фазового пространства.
Аноним 02/02/21 Втр 16:28:32 528049440
>>528040
Реализм протух еще 100 лет назад. В реальности я это никак не представляю.
Аноним 02/02/21 Втр 16:28:36 528050441
>>528048
А как ты представляешь себе "размерность фазового пространств"?
Аноним 02/02/21 Втр 16:29:26 528051442
>>528049
А как у тебя с крышей? Не течет?
Аноним 02/02/21 Втр 16:29:36 528052443
>>528050
Как кол-во переменных в функции.
Аноним 02/02/21 Втр 16:35:30 528053444
Аноним 02/02/21 Втр 16:36:19 528054445
>>528043
Частицы двигаются в своем маняпространстве, это пространство можно "спроецировать" на наше, однако эта "проекция" дважды(а иногда и более) покрывается наше пространство. Спин указывает на какую часть маняпростарства "проецируется" наше пространство.
Аноним 02/02/21 Втр 16:41:15 528055446
Аноним 02/02/21 Втр 16:42:08 528056447
>>528054
То исть никаких аналогий в нашем мире спину нет?
Аноним 02/02/21 Втр 16:43:44 528057448
>>528056
Только если сложные аналогии придумывать, там аналогии будут сложнее чем сами формулы тогда.
Аноним 02/02/21 Втр 16:44:42 528058449
>>528057
Жаль. Ну я тогда пошел.
Аноним 02/02/21 Втр 16:45:48 528059450
>>528056
Вращение вокруг своей оси. Земля вращается вокруг своей оси - это, можно сказать, "спин" Земли. Вот тебе аналогия. Вероятно, она грубоватая, но хоть какая-то.
Аноним 02/02/21 Втр 18:01:36 528063451
Аноним 02/02/21 Втр 19:13:59 528067452
>>528052
Вот ты и сломался.
Аноним 02/02/21 Втр 19:19:53 528068453
>>527985
>Может под неопределённым составом имелось в виду состав какого-нибудь источника света, лампочка например? Хотя да, и в лазерах разброс по длине волны присутствует.
Тут речь про само определение "единичный фотон".
Т.е. читая какой-то метод в котором рассказывают как получить "единичный фотон" возможно они имеют совсем не тот "единичный фотон" который хочешь ты, а какой-то "пакет".
Аноним 02/02/21 Втр 19:24:35 528069454
>>526777
Потому что не существует бесконечной точности.
Аноним 02/02/21 Втр 19:25:26 528070455
>>526754
>>526755
Жаль я забыл зачем мне это было нужно.
Аноним 02/02/21 Втр 20:41:06 528073456
Аноним 03/02/21 Срд 03:27:21 528088457
1Mn3XKTNVD8ImdB[...].gif 7161Кб, 500x500
500x500
Что такое волновая функция?
Аноним 03/02/21 Срд 06:48:54 528093458
Пока "а как ты себе это представляешь?"-шиз засирает тредю, реквестирую статейки с описанием/схемой устройств, которыми получают одиночные фотоны, ибо мало вразумительного нахожу в интернете среди кучи журнашлюшьего кала.
Аноним 03/02/21 Срд 06:56:22 528094459
>>528088
Функция сложной синусоиды
Аноним 03/02/21 Срд 08:17:23 528098460
Бля, зачем мне нужен был единичный фотон(((
Аноним 03/02/21 Срд 08:18:23 528099461
>>528093
Зачем тебе единичный фотон?
Аноним 03/02/21 Срд 08:29:47 528100462
А, вспомнил, чтобы доказать что у фотона вполне себе есть одно конркетное направление и размер, основываясь на реальных замерах, а не эти кукареки квантовошизикопетухов что он "находится сразу одновременно везде пока не попал в мишень, являясь областью вероятностей".
Аноним 03/02/21 Срд 08:37:05 528101463
>>528098
Может тебе нужен один очень энергичный фотон?
Энергией так на 100 ГэВ?
Аноним 03/02/21 Срд 08:39:20 528102464
>>528101
Ну да, хотел бы такой запулить в одного пидора

Вот только не знаю, может его бы даже не распидарасило, может он просто прошёл бы через него этот фотон даже не заметив этого пидора...
Аноним 03/02/21 Срд 09:43:48 528103465
>>528102
Несчастному придет конец. Рождение электрон-позитрон пар пар в электромагнитном поле атомных электронов его тушки и последующая аннигиляция с образованием еще фотонов и еще и еще. Настоящий ливень гамма-квантов прямо в его теле.
Аноним 03/02/21 Срд 10:49:26 528105466
Как занимались физикой до изобретения векторов?
Аноним 03/02/21 Срд 10:51:18 528106467
>>528103
Замечательно
Жаль длина свободного пробега такой свето-штучки в воздухе будет считанные миллиметры
Аноним 03/02/21 Срд 10:57:16 528107468
>>528099
На светоделительный куб его загнать
Аноним 03/02/21 Срд 10:57:35 528108469
Вот кстати вспомнил с чего начались все эти мои обдумывания про единичный фотон, про длину свободного пробега их в воздухе

Только не таких энергий а обычных, но разницы нет
Аноним 03/02/21 Срд 11:50:00 528114470
>>528100
Дегенерат, то, что любая квантовая частица, в том числе фотон, находится в суперпозиции до измерительного акта - это надежный факт, базирующийся на огромной массе экспериментальных данных. Твое голословное пуксреньканье, отрицающее факты, выглядит совершенно смехотворно. Не позорься.
Аноним 03/02/21 Срд 11:50:46 528115471
>>528093
Ты напиши не в гугле а в гугл академии. Там онли статьи научные, без кала.
Аноним 03/02/21 Срд 12:01:48 528118472
>>528114
А где в твоих пуках эти факты, манька?
Раз факты есть, зачем ты пукаешь пустым текстом без фактов?
Аноним 03/02/21 Срд 12:06:56 528119473
Аноним 03/02/21 Срд 12:25:31 528121474
>>528106
>Жаль длина свободного пробега такой свето-штучки в воздухе будет считанные миллиметры

Фотона с энергией 100 Гэв? Типа миллиметр воздуха защитит от гамма-квантов? Срочно пищи в Росатом, пусть свинцовую защиту снимают, хватает одного воздуха.
Аноним 03/02/21 Срд 13:04:53 528123475
SimplePeriodicT[...].png 130Кб, 1205x638
1205x638
Период полураспада флеровия-289 в русской википедии 80 с
Период полураспада флеровия-289 в английской википедии 1,9 с
Период полураспада флеровия-289 в немецкой википедии 2,7 с
Период полураспада флеровия-289 во французской википедии 2,6 с
Период полураспада флеровия-289 в японской википедии 2,6 с

Кто прав?
Аноним 03/02/21 Срд 13:05:01 528124476
>>528118
Самый легко проверяемый факт - существование неполяризованного света как такового.
Дифракция, интерференция и рассеивание - чисто волновые явления, которые нельзя описать через поток частиц.
Фотон выступает как частица в довольно узких местах - тепловое излучение и переизлучение атомов.(Атом кстати квантовая система)
Аноним 03/02/21 Срд 13:24:42 528125477
>>528124
Ты ему ничего не докажешь, ибо твои слова для него просто набор звуков. Он не знает математику и физику, а значит не сможет воспринимать информацию. Просто забей на него, может до него дойдет когда-нибудь в чём он ошибся, а может нет и он будет просто фриканутым. Это всё от лени + гордыни. Люди считают, что они достаточны умные, чтобы понять всякие сложные вещи, но при этом слишком ленивые или тупые, чтобы потратить время на изучение.
Аноним 03/02/21 Срд 14:46:50 528127478
>>528124
>чисто волновые явления
А кто спорит с волновой природой?
Речь о размере, у волны вполне есть размер.
Аноним 03/02/21 Срд 14:47:19 528128479
>>528125
Смачный разрыв квантовошизика.
Аноним 03/02/21 Срд 14:49:45 528129480
>>528127
И какой размер у синусоиды вправо и влево?
Аноним 03/02/21 Срд 14:49:51 528130481
>>528121
А какая длина пробега?

Снимать защиту не поможет, т.к. он после первого поглощения разобъётся на ливень сильно меньше, но тоже очень энергетических частиц, ты же(или другой анон) сам писал.
Речь о том что его как выстрел из снайперки не получится использовать.
Аноним 03/02/21 Срд 14:53:29 528131482
>>528129
Это и пытаюсь выяснить.

Мне думается что фотон некое овальное возмущение электромагнитного поля.
Объмное естественно
Имеющее ширину немного меньше длины
Аноним 03/02/21 Срд 14:57:50 528132483
>>528131
По определению, если взять какой-то объем V и туда запихнуть один фотон, то он будет представлять из себя синусоиду электромагнитного поля от одной стенки до другой. Это если он обладает конкретной энергией.
Аноним 03/02/21 Срд 15:25:54 528133484
>>528131
А ты хоть вообще понимаешь, что такое поле, тем более что его возбуждение?

И тебе уже говорили, что фотон нельзя представить шариком. И вообще он размазан по всей вселенной.

Длина волны это характера волны и она зависит от внешних условий.
Аноним 03/02/21 Срд 15:43:59 528135485
>>528133
>И вообще он размазан по всей вселенной.
Ну вот, опять начинается эта шиза

А я и хочу разобраться, были ши эксперименты, подтверждающие это.
Для этого и нужна штука, выпускающая один фотон.
До того как я про это всё дело забыл мы остановились о том что терминология немного отличается от реальности, и приборы/установки экспериментальные, которые считаются что выпускают 1 фотон, на самом деле выпускают не 1 фотон, а "пакет" ты говоришь
Это не подходит.
Аноним 03/02/21 Срд 15:44:40 528136486
>>528118
Суперпозиция частицы - это факт, чмоня. Давай опять пуксренькай "вы фсе врети".
Аноним 03/02/21 Срд 15:45:50 528137487
>>528136
Приведи пример, опиши эксперимент который это подтверждает.
Аноним 03/02/21 Срд 15:58:50 528138488
>>528136
Опыт штерна-герлахта подтверждает, что состояние магнитного момента иона серебра и просто частиц со спинпроекцияом 1/2 описывается суперпозицией состояний " на выделенную ось 1/2" и "проекция на выделенную ось - 1/2" со всеми вытекающими, а никак не вектором в трёхмерном пространстве.
Аноним 03/02/21 Срд 15:59:44 528139489
Аноним 03/02/21 Срд 16:15:47 528140490
>>528130
>А какая длина пробега?
А что фотон 100 Гэв такой пробегает миллиметр в воздухе и РАЗ и внезапно исчезает?
Аноним 03/02/21 Срд 16:28:09 528141491
>>528135
>А я и хочу разобраться, были ши эксперименты, подтверждающие это.
Чем тебе не нравится опыты с поляризаторами? Прекрасный масштабируемый эксперимент иллюстрирующий квантовую суперпозицию.
Или дифракция рентгеновский лучей на кристаллах? Опыт подразумевает набор статистки со вспыхивающими точками на экране.
Многие астрономические приборы регистрируют единичные фотоны.
Кстати, ты в курсе как работают ПЗС матрицы?

Так же и касаемо генерации фотонов. При генерации фотонов в цепи питания создается особый шум, возникающий из-за дискретности излучения.

>которые считаются что выпускают 1 фотон, на самом деле выпускают не 1 фотон, а "пакет" ты говоришь
Ты специально все путаешь?
Тебе уже говорили, что фотон это совокупность волн, которые рандомно выпадают при измерении. Каждая волна сама по себе уже не является фотоном, это блядь просто классическая эм волна, которая имеет вероятность задетектироваться.

Проблема генерация одного фотона вытекает из трудности приготовления квантовых состояний. Они очень чувствительны к внешним воздействий. Есть вероятность просто потерять фотон, поэтому с одиночными фотонами никто не работает.
Аноним 03/02/21 Срд 17:04:37 528142492
>>528141
>Тебе уже говорили, что фотон это совокупность волн
Э нет, фотон это и элементарная частица и волна.
мимо
Аноним 03/02/21 Срд 17:06:15 528143493
>>528141
>особый шум
Лолшто?
Аноним 03/02/21 Срд 17:19:07 528145494
>>528142
Лол, почти любой учебник или курс лекций по квантовой механики начинается с выкидывания термина частица из употребление и введение понятия кванта.
>>528143
Я точно не помню, но по своей природе он похож на дробовой шум.
Аноним 03/02/21 Срд 17:33:21 528147495
Аноним 03/02/21 Срд 19:14:54 528149496
>>528140
А воздух у тебя что ваккум?
Аноним 03/02/21 Срд 19:17:07 528150497
>>528141
>Опыт подразумевает набор статистки со вспыхивающими точками на экране.
Так ты же не можешь выпустить один фотон, какую статистику ты собираешь, лалка?
Аноним 03/02/21 Срд 19:42:40 528151498
>>528150
С рентгеновской трубкой довольно легко добиться испускания одиночных фотонов.
А радиоизотопные источники вообще сугубо штучные.
Аноним 03/02/21 Срд 19:50:41 528153499
>>528151
>А радиоизотопные
Ты отсортировал один атом?

>С рентгеновской трубкой довольно легко добиться испускания одиночных фотонов.
Так одиночных или пакет?
Аноним 03/02/21 Срд 20:07:22 528154500
>>528153
>Ты отсортировал один атом?
Зачем? Радиоактивный распад так устроен, что ядро за каждую секунду имеет вероятность совершить распад с излучение кванта. Можно подобрать такие изотопы, что генерили в среднем один квант в секунду.

>Так одиночных или пакет?
Именно строго один квант. В современной КТ дошли до почти полного отслеживания одиночный квантов.
Что ты прицепился к слово пакет?
Аноним 03/02/21 Срд 20:13:04 528155501
>>528153
Один квант может быть пакетом. Это не противоречащие друг другу вещи.
Аноним 03/02/21 Срд 20:18:28 528156502
>>528154
>что генерили в среднем
>ой, а аткуда это у нас в опытах появилась неопределенность, и всё почему-то можно посчитать только среднестатистическии... да явно квантовая неопределенность есть истинная природа мира!!!
Аноним 03/02/21 Срд 20:21:28 528157503
>>528154
>Что ты прицепился к слово пакет?
Так а ты пишучи вот это
>Именно строго один квант. В современной КТ дошли до почти полного отслеживания одиночный квантов.
Имеешь ввиду пакет?
Прицепился потому что насколько понял, общепринятое определение "один квант"="пакет", а не "истинно одиночный фотон".
Кстати о каком ты кванте? Мы же о фотонах.

А что такое "истинно одиночный фотон"? В чём отличие от пакета?
Аноним 03/02/21 Срд 20:21:47 528158504
>>528156
Предложи модель более лучшую прогностическую модель.
Аноним 03/02/21 Срд 20:22:05 528159505
Аноним 03/02/21 Срд 20:26:15 528161506
>>528158
Сначала нужны эксперименты с истинно единичными фотонами, а не "в среднем 1 фотон, почему же можно посчитать только среднестатистическии..."
Аноним 03/02/21 Срд 20:44:58 528162507
>>528157
По определению фотон это квант(порция) эм поля.
"Истинно одиночный фотон" это просто строго один квант.
Каждый квант можно представить в виде волнового пакета - совокупности эм волн, который выпадут нам при измерение этого фотона.
Сам по себе фотон не интересен, важен только его волновой пакет.
В конечном случае фотон представляется как щелчок детектора или вспышка на экране, сигнализирующая о том, что через нее прошла некоторая эм волна. Характеристики волны определяются волновым пакетом. Выпадает только одна волна из всего множества, другие просто уже никак не будут реализованы в нашей реальности.

Фотоны не взаимодействуют с друг другом и благодаря этому статистически их удобно считать не по одиночке, а сразу большим количеством штук, заменяя неким средним. В добавок в конечных измерениях мы все равно не имеем дело с одиночными квантами, а с их большим количеством.
Аноним 03/02/21 Срд 21:15:01 528163508
>>528159
Понятное дело есть. Но суперпозиция состояний поля с разной энергией но с 1 фотоном, вполне себе квант.
Аноним 04/02/21 Чтв 02:02:42 528165509
>>528162
>По определению фотон это квант(порция) эм поля.
Не совсем. По определению фотон это частица и одновременно волна.
Аноним 04/02/21 Чтв 02:30:50 528167510
>>528165
Чел, давай без троллинга тупостью. Фразами "частица и волна" пользуются только в научпоп высерах.
Аноним 04/02/21 Чтв 04:40:23 528170511
>>528167
и общей физике - очень удобно не считать многомерную волновую функцию в случаях, когда можно пользоваться дуализмом) например при расчете фотоэффекта.
Аноним 04/02/21 Чтв 05:54:07 528175512
>>528163
>Но суперпозиция состояний поля
Так нету никакой суперпозиции, уже определились что экспериментируют с "в среднем 1 фотоном", в лучшем случае с "пакетом".
Будет один фотон-не будет никакой суперпозиции.
Аноним 04/02/21 Чтв 05:55:42 528177513
>>528162
>Каждый квант можно представить в виде волнового пакета
А можно этот пакет представить в качестве множества квантов.
Значит этот пакет нельзя считать одним фотоном.
Аноним 04/02/21 Чтв 05:59:53 528178514
>>528162
>В конечном случае фотон представляется как щелчок детектора или вспышка на экране, сигнализирующая о том, что через нее прошла некоторая эм волна. Характеристики волны определяются волновым пакетом. Выпадает только одна волна из всего множества, другие просто уже никак не будут реализованы в нашей реальности.
Слишком общее представление, "щелчок детектора" или "вспышка на экране" это совсем шлак для детей, сорри
Давай про фундаментальный принцип работы детектора
Если пакет это всё-таки сразу несколько волн, принцип работы детектора позволил бы задетектить их все, по очереди, или после детектирования одной, он бы уже "был взведён" и на остальные из "пакета" не отреагировал бы и они просто пролетели бы мимо, или были поглощены но в тепло, а не в сигнал детектора?
Аноним 04/02/21 Чтв 06:02:16 528179515
Вот когда одиг атом своим электроном поглощает фотон, он поглощает всего 1 фотон, или сразу весь пакет?
Аноним 04/02/21 Чтв 06:11:15 528180516
>>528162
Так подожди, что всё-таки значит пакет?
Если рассматривать фотон именно как волну, с частотой определённой, строго одной частоты
То пакет это получается якобы вероятность нахождения в месте и во времени где-то относительно рядом(что детектят приборы и по этому принято считать как один фотон) нескольких волн разной частоты?
А вероятностный потому что:
или источник высерает не одну волну одной частоты, а сразу несколько разных частот
или высирает с разной вероятностью каждый раз не строго одну, а 0-2 ,0-0-3, 0-0-0-4 и т.д. и тоже походу разных частот
или детектор детектит влетающие в него волны не строго каждый раз, а с вероятностью срабатывания
или показывает измеренную длину волны с какой-то ошибкой
или комбинация любых нескольких этих вариантов.
Аноним 04/02/21 Чтв 06:15:05 528181517
>>528177
>А можно этот пакет представить в качестве множества квантов.
Зависит от пакета.
>>528178
Пакет это не "несколько фотонов". Стандартная путаница у людей, не знающих квантовую механику в том, что они путают классическую вероятность и квантовую.
Аноним 04/02/21 Чтв 06:22:32 528182518
>>528181
>они путают классическую вероятность и квантовую
Так это же один хер проверяется приборами.
С вероятностью срабатывания.
Аноним 04/02/21 Чтв 07:22:21 528189519
>>528181
>путают классическую и квантовую вероятность
из-за таких как ты и путают. про измерения рассказывают, а про то, что можно что-то кроме кол-ва фотонов не рассказывают.
Аноним 04/02/21 Чтв 07:37:54 528192520
>>528182
смотри в чем разница. если у фотона суперпозиция 50\50 поляризации вертикальной и горизонтальной, то есть положение фильтра, когда он 100% пройдет через него.
если у нас СМЕСЬ 50\50, то он всегда будет поглощаться с шансом 50\50, в этом и разница.
Аноним 04/02/21 Чтв 09:32:15 528199521
>>528192
>если у нас СМЕСЬ 50\50
Так у тебя же прибор испчксающий фотоны выплёвывает не строго такие как ты хочешь, а с вероятностью какие-то там.
Аноним 04/02/21 Чтв 09:36:47 528201522
Wienslaw.svg.png 35Кб, 440x367
440x367
Слушьте, а закон смещения вина
Он только из-за того что нижняя шкала графика, длины волны, равномерна, ну по длине волны, а ЭНЕРГИЯ излучения/фотонов не прямо зависит от длины волны?
Аноним 04/02/21 Чтв 09:46:11 528202523
Вообще с чего я всё это начал.
Хотя про эксперименты с истинно одиночным фотоном это тоже интересно,
но
Похер на самом деле, истинно 1 фотон или нет, хоть пакет, хоть десяток.
Главное достаточно малое количество их
Фотон поглощается атомом/электроном атома или едром, при "ПОПАДАНИИ", это факт.
Воздух состоит из атомов и молекул, некой плотности количесвенной на объём.
Значит, летя через воздух, фотон, или пакет, или 10 фотонов, через какое-то расстояние пролёта встретят на своём пути атом воздуха, поглотятся и переизлучатся.
Так? Это вроде тупо факт.
Почему тогда говорите нету смысла говорить о длине свободного пробега фотона в воздухе?
Поставь лист стали перед фонариком на расстоянии 1 см от него и длина свободного пробега фотонв из фонарика будет 1 см, ебать.
Так ведь нету разницы, ЛИСТ СТАЛИ или воздух.
Буддет разница только в длине свободного пробега.
Аноним 04/02/21 Чтв 10:59:12 528205524
Сап, надеюсь. вопрос по треду, купил с рук подержанную трубку гейгера сбм 20, у неё корпус в свинец обёрнут - может ли от неё по каким-то причинам фонить радиацией?
Аноним 04/02/21 Чтв 12:19:50 528219525
>>528177
>А можно этот пакет представить в качестве множества квантов.
Вообще нет. С несколькими квантами уже другие картины возникают.
>>528178
И прибор детектит только одну единственную волну из всего пакета. Другие исчезают из реальности. И нет способа это обойти.
>>528180
>Если рассматривать фотон именно как волну, с частотой определённой, строго одной частоты
Тебе уже сказали, почему это не верно.
А источник вероятность вытекает из того, что суперпозицию волн нельзя проецировать в реальность разом.
Аноним 04/02/21 Чтв 12:38:01 528224526
>>528219
>Другие исчезают из реальности
Это шиза
Они просто или пролетели, или их там и не было.
Аноним 04/02/21 Чтв 12:39:10 528225527
>>528219
>что суперпозицию волн нельзя
Ну какая суперпозиция, ты пока не научился испускать строго одинаковые фотоны, а только разные с какой-то вероятностью.
Аноним 04/02/21 Чтв 12:41:22 528226528
>>528202
Помимо поглощения есть и рассеивание, ведущий к тому, что фотон детектируется там где не должен.
И если мы направим на кусок вещества(стекло например) условные 1000 фотонов: 2 поглотится, 80 зеркально отразятся, остальные пройдут насквозь и немного "уйдут" в сторону.
В случае воздуха фотоны практически не поглощаются, вместо этого испытывают рассеивание и их "уводит" с прямой.
В случае металлов несколько сложнее, из-за особенности рассеивания фотоны почти не проходят дальше поверхности. Что 1мм фольга, что 1 см лист одно и тоже.
Аноним 04/02/21 Чтв 12:47:03 528229529
>>528224
То что это не интуитивно и противоречит твоим бытовым представлением не значит, что этого не должно быть.
Многим не нравится коллапс волновой функции, но с этим сделать ничего не могут.
>>528225
Одинаковые фотоны нельзя принципе излучить.
Генераторы одиночных фотонов уже давно сделали, но они нахуй кому нужны.
Аноним 04/02/21 Чтв 13:21:41 528236530
>>528226
>В случае воздуха фотоны практически не поглощаются, вместо этого испытывают рассеивание и их "уводит" с прямой.
Ну так природа этого рассеивания-поглощение и переизлучение.

Я об этом же и толкую
Ведь вообще самая изначальная суть моего вопроса, раз фотоны движутся через воздух, через среду, по средствам поглощения и переизлучения
А переизлучение вроде происходит в случайном направлении
То какого хуя свет так долго и ровно может через воздух идти
Учитывая что длина среднего свободного пробега фотона в воздухе наверное очень маленькая, то воздух должен почти сразу равномерно рассеивать попавший в него свет.

Поэтому я и пытаюсь узнать среднюю длину свободного пробега фотона в воздухе.

Ну и на скорость света это должно сильно влиять, а не /1.03 или сколько там коэффициент скорости света в воздухе при н.у. в отличии от вакуума.
Аноним 04/02/21 Чтв 13:23:09 528237531
>>528229
>Генераторы одиночных фотонов уже давно сделали
Которые излучают не одиночные фотоны, а 0-2, или 0-0-3, или 0-0-0-4
Или "пакет"
Аноним 04/02/21 Чтв 13:25:17 528238532
>>528229
>То что это не интуитивно и противоречит твоим бытовым представлением не значит, что этого не должно быть.
Причём тут это, ты не можешь доказать это
>>528219
>Другие исчезают из реальности.
А значит это шиза и маняфантазия.
Аноним 04/02/21 Чтв 13:28:57 528240533
Кстати, а какая магия в опыте с 3 фильтрами?

Ведь никто не говорит как работают эти фильтры
Они просто "поворачивают" фотоны
Чем на меньший угол и плавнее "поворачивают", тем больше вероятность что фотоны пройдут.
Поворачивают сразу на 90 градусов двумя фильтрами-почти ни один не может пройти.
Поворачивают сначала на 45 потом ещё на 45 3мя фильтрами-почти все проходят.
Чё тут магического?
Аноним 04/02/21 Чтв 14:43:42 528254534
>>528237
Какие еще 0-2 и 0-0-3?
Ты контролируемо возбуждаешь систему(пинаешь возбужденный атом), она мгновенно скачкообразно меняться, при этом в другом месте щелкает детектор. впихивает экран. засвечивается фотоэмульсия или в фотодетекторе возник ток. Это соответствует испусканию и детектции одного фотона. При достаточно чувствительности легко заменить, что этот скачек единственный: один пинок - одна единственная вспышка. Далее уже довериваешь приблуды в виде щелей поляризаторов для определения уже волновых свойств фотона.
>>528238
Мне тебя за ручку вести в лабораторию. чтоб доказать?
Аноним 04/02/21 Чтв 15:12:26 528261535
>>528236
Рассевание это одно, а поглощение или переизучение это совершенно другое. Это совершенно различные явления.
Первое чисто волновая оптика, у них своя модель.
Второе чисто квантовая оптика, у другая модель, которая радикально отличается от первой.

Преломление, дисперсия и все когерентные рассеивания объясняются без моделей поглощения-рассеивания. И более того, если ты будешь исходит из которого. что фотоны в веществе постоянно переизлучаются, то получишь нерабочую модель. Вместо этого рассчитывают прохождение волнового фронта.
Аноним 04/02/21 Чтв 16:48:55 528274536
От кого из родителей передается интеллект ребенку?
Аноним 04/02/21 Чтв 18:08:13 528284537
>>528274
Интеллект - это не триппер, чтобы передаваться от конкретного носителя. Это сложное комплексное явление, которое обусловлено совокупностью генетических и средовых факторов. Среди генетических факторов роль играют гены обоих родителей.
Аноним 05/02/21 Птн 09:33:29 528329538
>>528261
>Рассевание это одно, а поглощение или переизучение это совершенно другое.
Ты скозал?
Аноним 05/02/21 Птн 09:36:22 528330539
>>528254
>Мне тебя за ручку вести в лабораторию. чтоб доказать?
Зачем, если там не могут выпускать строго один фотон?

>Ты контролируемо возбуждаешь систему
С какой точностью?

>она мгновенно скачкообразно меняться, при этом в другом месте щелкает детектор. впихивает экран
Ты будто просто методичку для школьников класса 8-го заученную повторяешь.
Аноним 05/02/21 Птн 18:05:34 528364540
2021-02-0422-46[...].png 67Кб, 863x862
863x862
Правильно ли я понимаю, что люди, которые любят милых девушек - педофилы?
Аноним 05/02/21 Птн 21:06:18 528376541
Есть ли термин для обозначения таких феноменов как фингербокс, желемыш или того же самосбора? Я пытался искать, из близкого нагуглил только заумь, но это не то.
Аноним 05/02/21 Птн 21:14:59 528377542
>>528364
Нет, это норма. Точно так же мы не можем считать людей, которым нравятся зрелые девушки, геронтофилами.
Аноним 05/02/21 Птн 21:29:08 528382543
Что такое мысль?
Аноним 05/02/21 Птн 22:38:52 528384544
>>528377
Не согласен. Милфы - это скорее про инцест между матерью и сыном. (потому что человек, который хочет трахнуть зрелую женщину, скорее всего, просто ищет кого-то похожую на мать. Либо ищет просто опытную женщину, но это другой случай). Так же и здесь, если тебя привлекает вся эта милота и начинаешь трахать такую тян, то это психологическая педофилия, имхо. Но тут нужно разграничивать как-то, когда это просто дело вкуса, а когда (даже не знаю как выразиться правильно) ты имеешь психологическую зависимость или что-то типо того.
Аноним 05/02/21 Птн 22:55:26 528385545
>>528330
Тебе сказали, что с помощью фильтра нельзя получить строго один фотон. Но никто не говорил про другие способы получения.
Аноним 06/02/21 Суб 00:50:36 528390546
>>528382
Поток, движение, перемещение частиц.
Аноним 06/02/21 Суб 00:52:27 528391547
>>528385
Это не мне, и другие способы "получить один фотон" тут говорили множество раз, никто чётко не сказал и не доказал что там получается строго один фотон.
"В среднем один фотон"-да.
"Пакет"-да.
Один фотон-нет.
Аноним 06/02/21 Суб 01:01:05 528392548
>>528364
>милых
>педофилы
Тупая мысль, дети зачастую крайне мерзкие бывают.
Аноним 06/02/21 Суб 01:01:46 528393549
>>528376
Говно, даунизм, имбицильность, дегенератность.
Аноним 06/02/21 Суб 02:35:00 528396550
>>528391
Ох бля, понимал бы ты еще что такое волновой пакет.
Аноним 06/02/21 Суб 05:27:23 528398551
>>528364
Почти все женщины детского и репродуктивного возраста экстремально милые с биологической т.з. И нет никаких сомнений, что охотники-собиратели (а также люди Бронзового века) ебали и ебут детей всех полов. И овец. Но милота это один из стимулов. Сверхстимул это половая зрелость.
Социализированные люди игнорируют стимулы и реагируют на сверхстимулы. Они сразу не бьют ебало в ответ на агрессивный тон и зрительный контакт, а ждут, пока их толкнут. У педофилов (и многих убийц) или нет нормальной социализации, или дефектные мозги.
Аноним 06/02/21 Суб 05:38:14 528399552
download.png 4Кб, 225x225
225x225
>>527834
Бамп.
Как узнать, насколько "люди тупые"? И в плане общей эрудиции (которая требует внимания и любопытства), и допустим, способности к простейшим логическим операциям.
Допустим, 80% моих одноклассников/одногруппников на самом деле не умели пользоваться логическими операциями (введение в программирование в школе, формальная логика в вузе). Они не были умственно-отсталыми и не имели СДВГ, значит, как нормальные люди, могли бы усвоить эту хуйню. Но не усвоили. Они тупые? Но незнание не помешало им получить дипломы (а знание никак не пригодилось бы, ведь у них нет интересов, которые привели бы их в соотв. области). Они не тупые?
Аноним 06/02/21 Суб 06:09:31 528401553
Аноним 06/02/21 Суб 06:40:36 528403554
>>528376
Странно, я погуглил "хронопы" + "сепульки", и получил один результат, страницу форума. На tvtropes вроде тоже ничего нет.
Значит это приём без специального названия, просто выдуманные термины, в обиход их ввелии дадисты, сюррелисты и прочие пидорасы.
Аноним 06/02/21 Суб 11:37:07 528406555
>>528398
Но ведь отчасти получается, что ты хочешь трахнуть ребенка, пусть и сексуального.(имею ввиду людей, которые таки реагируют на стимул "милоты") Это плохо или хорошо для психики?
Аноним 06/02/21 Суб 12:49:07 528408556
>>528390
Это довольно общее описание. А конкретнее можно?
Аноним 06/02/21 Суб 13:28:49 528411557
image.png 80Кб, 1620x273
1620x273
Свет всегда двигается по прямой? В курсе школьной физики говорили, что да. Сейчас наткнулся на пикрил. Что в итоге?
Аноним 06/02/21 Суб 13:50:50 528413558
изображение.png 804Кб, 677x853
677x853
изображение.png 290Кб, 800x516
800x516
Скорее всего вопрос тупой для местных анонов.

Есть лазерная ПРО.
Есть боевые блоки.

Вопрос:
Когда боевые блоки входят в атмосферу, их окутывает облако плазмы. Так вот, как себя поведёт лазер в таких условиях? Сможет ли он пробить как толщу атмосферы до блока, так и само плазменное облако вокруг боеголовки?
И заодно, если не трудно, не подскажите время падения боевого блока?

(просто спорю с человеком, он утверждает что лазерное ПРО уже с 2004 года решило проблему лазера в плазме. мои же знания по физике говорят что лазер в плазме не распространяется и ему надо буквально пробивать дорогу, что при большой скорости и теплозащите боевого блока не сильно ему и поможет(лазеру))
Аноним 06/02/21 Суб 17:10:01 528430559
>>528413
>подскажите время падения боевого блока?
Время падения из апогея или время от старта до падения?
Аноним 06/02/21 Суб 17:20:08 528431560
>>528411
Это уже квантовая электродинамика, там возникают тонкости и всякое такое. А так да, свет движется по прямой. Почитай книгу Фейнмана КЭД или странная теория света и вещества.
Аноним 06/02/21 Суб 18:51:15 528434561
>>528430
Из апогея.
Там спор - сколько успеет сделать вредного лазер для боевого блока МБР во время падения оного на цель, а для этого надо знать время падения.

Ну, и если не трудно, можешь ответить про плазму и лазер. Всё же вспоминая мои уроки физики - плазма - это то что хорошо так убивает лазерный луч.
Аноним 06/02/21 Суб 19:25:38 528436562
>>528434
Около 7-10 минут, но может быть гораздо меньше если ракету пускали из подводной лодки с меньшего расстояния.
Аноним 06/02/21 Суб 19:38:31 528438563
>>528436
Хм. С апогея она падает примерно 7 минут? Ого, это долго.
А меньше если с подлодки - за счёт меньшей высоты полёта?
Аноним 06/02/21 Суб 20:01:47 528443564
>>528438
С подлодки траектория в целом короче.
Аноним 06/02/21 Суб 20:22:37 528446565
>>528443
Ок. Спасибо. Теперь буду ждать ответа по плазме и лазеру.
Аноним 07/02/21 Вск 03:48:46 528456566
>>528406
Дети ИРЛ должны вызывать у тебя легкое отвращение (как мужчины у гетеросексуальных мужчин, вне зависимости от их красоты). Если вызывают возбуждение, то поздравляю, ты педофил/пидорас.
Аноним 07/02/21 Вск 09:17:32 528459567
>>528456
Ты меня не понял. Я имел ввиду не маленьких детей, а тян, которые выглядят мило и тебя это привлекает. А если тебя это привлекает, помимо сексуальности оной, то получается как будто бы ты трахаешь ребенка.(в особенности, если у тебя зависимость от этого)
Аноним 07/02/21 Вск 16:26:27 528469568
>>528456
Не, просто мужики сами по себе мерзкие, большинство даже у женщин легкое отвращение вызывают.
мимопидор
Аноним 07/02/21 Вск 16:59:14 528470569
image.png 48Кб, 186x207
186x207
Мне в голову пришла просто гениальная идея: вместо водяного охлаждения в компы ебануть РТУТНОЕ. Просто представьте, каких коэффициентов теплоотдачи можно достигнуть на жидком металле.

Как вам идея? Не будет ли коррозии алюминиевых радиаторов под воздействием ртути?
Аноним 07/02/21 Вск 17:07:37 528471570
>>528470
Будет, и очень, очень быстрая.
Аноним 07/02/21 Вск 17:21:29 528473571
>>528471
Тогда нужно будет делать радиаторы из коррозионно-стойкой стали.
Аноним 07/02/21 Вск 17:24:27 528475572
>>528473
Создается впечатление, будто ты понимаешь все процессы, которые делают идею неудачной, но просто продолжаешь траллить...
Аноним 07/02/21 Вск 17:28:32 528476573
>>528475
Я что-то да знаю о ртути в ипостаси теплоносителя и проблемах с ней, тем не менее других жидких металлов в голову не приходит. Тот же натрий слишком активный: мало кому понравится факел на столе.
Аноним 07/02/21 Вск 18:02:45 528477574
>>528476
Коррозия ртутью и коррозия воздухом это как теплое и мягкое. Нержавейка тебя не спасет.
Аноним 07/02/21 Вск 19:44:33 528485575
>>528477
Процессы там разные, но тем не менее это фиксится легированной всякими титанами-ваннадиями сталями.
Аноним 07/02/21 Вск 23:04:49 528493576
>>528469
Геям нравится мужики или хуй сосать/в зад пороться?
Аноним 07/02/21 Вск 23:54:28 528501577
>>528485
Ртуть банально не допустят до рабов. Ртутные градусники не зря изъяли. Cейчас изымают вообще все потенциально опасное, что не положено рaбу. Скоро и металлических ножей и вилок у тебя не будет. И это даже к учетной рабочей технике гос. служащих относятся, там тоже не станут использовать потенциальный способ отравиться.
А хороших решений вообще миллионы, использоваться они, конечно же, не будут. Ну, нет выгоды в том, чтобы сыч смог запускать киберпук 2077 при температуре проца 25°.

мимо
Аноним 08/02/21 Пнд 22:11:42 528552578
Привет, анонимус.
Я хочу провести беседу в войсчате дискорда с трансляцией. Суть такова:
Диспут. Двое людей, или равные по количеству участников группы, излагают свои мнения относительно существования/отсутствия Бога безотносительно каких-либо ссылок на научные или религиозные работы. Чистая философия.
Правила довольно простые (могут измениться, окончательные правила будут высланы участникам заранее):
- Запрещены любые оскорбления, в том числе "верун", "аметист" и подобные. Должно быть уважительное отношение к оппоненту.
- Мат запрещен если он используется для оскорбления оппонента или его позиции.
- Разделение дискуссии по времени. Типа вступительная часть, где каждый излагает за установленное время свою позицию, потом блиц-опрос, потом вопросы друг другу и так далее.

Прошу желающих участвовать написать мне на почту, надо будет пройти небольшой опрос:
empathy@nightmail.ru

Предположительное время проведения диспута - следующая пятница или суббота вечером.

На троллинг не отвечаю, любые сообщения в треде кроме этого - не мои. Все вопросы задавайте по почте.
Спасибо за внимание.
Аноним 08/02/21 Пнд 22:17:33 528553579
>>528470
>>528476
>других жидких металлов
Г А Л Л И Й
А
Л
Л
И
Й
Аноним 09/02/21 Втр 00:36:07 528562580
>>528398
> У педофилов (и многих убийц) или нет нормальной социализации, или дефектные мозги
Или они просто делают что хотят.
Аноним 09/02/21 Втр 00:36:59 528563581
>>528396
Ты уже говорил, волновой пакет это "не можем измерить и/или выпустить прибором строго 1 фотон."
Аноним 09/02/21 Втр 00:37:38 528564582
Аноним 09/02/21 Втр 00:42:45 528565583
>>528563
Нет нет нет... волновой пакет может быть и таким, что там будет всего один фотон. У тебя может быть суперпозиция состояний поля с одним фотоном, просто у этих состояний будет различная энергия (разная длина волны). И ты будешь, например, повторять эксперимент, а у тебя будут ловиться фотоны то 555.5 нм, то 555.4 нм, то 555.6 нм и т.д. При этом будет лететь не смесь фотонов, а просто фотон как бы "не определился какая у него частота" поэтому он размазан в импульсе своём.
Аноним 09/02/21 Втр 21:21:19 528702584
0f2.png 70Кб, 532x563
532x563
Аноним 10/02/21 Срд 20:32:33 528808585
У меня вопрос по экономике.
Я слышал, что во времена нэпа можно было купить облигации государственного займа. Вроде как и государству помог, и копейку заработал.
Сейчас есть биржи, акции. Облигации тоже есть, конечно.
Сейчас акции обычно покупают, чтоб обогатиться. Трейдеры.
Продать подороже, или дивиденды получить, или более сложные инструменты фин рынка.

А меня интересует другая сторона.
Если я покупаю акции компании - ей от этого ни плохо, ни хорошо. Никак. Просто сменился владелец доли.
А вот если я хочу помочь компании - дать ей деньги в долг под нулевой процент. Как это сделать?
Хочу дать в долг металлургам 10000 рублей. И через год получить 10000 рублей. Хочу, чтоб они на 10000 рублей привлечённых инвестиций модернизировали, или расширили производство.
Аноним 10/02/21 Срд 21:45:00 528811586
>>528808
Лично с металлургами договаривайся.
Аноним 13/02/21 Суб 10:25:54 528983587
Почему законы Вселенной таковы, каковы есть?
Аноним 13/02/21 Суб 14:16:17 529004588
>>528983
А какими им еще быть?
Аноним 13/02/21 Суб 22:03:58 529029589
Аноним 13/02/21 Суб 22:31:32 529032590
Аноним 14/02/21 Вск 07:45:55 529057591
>>529032
Ладно, я хотел сказать, что отсутствие у меня альтернативных предложений по поводу законов Вселенной не отменяет сути моего вопроса.
Аноним 14/02/21 Вск 12:21:14 529064592
>>529057
А в чём вопрос то?
Аноним 26/02/21 Птн 15:36:59 529746593
>>528983
>Почему законы Вселенной
А есть ли они вообще?
Аноним 28/02/21 Вск 11:10:05 529952594
Итак, есть вопрос по математике.
Есть два фактора влияющих на результат. Первые два фактора можно выразить уравнениями. Надо предсказать результат
Например, имеем мощность и вес автомобиля, как первые 2 фактора, которые выражены уравнениями. Нужно составить такую систему уравнений, что бы предсказать скорость машины. Надо составить что-то универсальное, что бы подставлять любые значения для конкретных параметров
Как такое сделать?
Аноним 28/02/21 Вск 18:29:20 529995595
>>529952
За предсказаниями к бабке ходить надо, а не уравнения составлять
Аноним 29/03/21 Пнд 18:57:39 532140596
Есть ли инстинкты у человека?
Настройки X
Ответить в тред X
15000
Макс объем: 40Mб, макс кол-во файлов: 4
Кликни/брось файл/ctrl-v
Стикеры X
Избранное / Топ тредов